Download as pdf or txt
Download as pdf or txt
You are on page 1of 91

Get more from: MplusX Qbank (https://member.mplusx.

com/)
Facebook Page: (https://www.facebook.com/mplusxqbank/)

A 65 years old Pt presented by light headiness, and palpitation, Examination was clinically
free, he’s on perindopril, Eltroxin, verapamil, antidiabetic medications. ECG (1st degree
heart block). What’s the next appropriate treatment?

A. Increase Eltroxin
B. Stop verapamil
C. Stop perindopril
D. Add digoxin

Couple come for infertility problem for the last 12 months. On testing examination and tests
of the female are unremarkable. Male has azoospermia. And bilateral absence of vas
deferens. Which of the following is describes the cause of infertility?
a. gonad hypertrophy
b. testicle cancer
c. Klinefelters
d. cystic fibrosis

Old lady with history of appendicectomy & cholecystectomy presented with 3 wk history of
abd pain, distension. xray shows dilated ascending colon, transverse colon & descending
colon, no air. Abd is soft tender. Rectum is empty.
A- Sigmoid volvulous?
B- Adhesive IO
C- CA Sigmoid

aA man had been on anti-Parkinson drug for 9 years. Recently, his wife complains of
psychosis and was prescribed 10mg Haloperidol. After two weeks, wife complains of
abnormal movement and inability to feed himself. What to do next.
A) decrease dose of anti-Parkinson drug.
B) increase dose of anti-Parkinson drug
C) decrease dose of Haloperidol
D) increase dose of Haloperidol
E) continue same treatment

An old man with acute onset pain in the lower back with urinary symptoms. significant
weight loss. On DRE, irregular prostate +. What is your appropriate investigation to lead you
to diagnosis?
a. chest x-ray
b. CT spine
c. TRUS

AfraTafreeh.com
Get more from: MplusX Qbank (https://member.mplusx.com/)
Facebook Page: (https://www.facebook.com/mplusxqbank/)

d. bladder scan
e. PSA

Woman around 30-35 years of age attends for Pap smear, Her previous smears are all normal.
She doesn’t have sexual activity in last two years. What’s the reason of testing chlamydia in
her?
A. chlamydia is asymptomatic
B. chlamydia can cause infertility
C. Both partners could be asymptomatic
D. No need for screening

Family history of father and paternal aunt died at 50 years of age. Patient complaint of wide
based gait and verbal dysfunction. What is your next management?
a. Genetic counselling
b. Lumbar puncture
c. Serum copper and ceruloplasmin
d. MRI
e. CT head

A young man has a syncopal attack while weight lifting. He has had similar episodes twice
before. His father died of cardiac disease. What is the next most appropriate management in
diagnosis?

a) Holter monitor

b) EKG

c) Stress test

d) BP in supine & lying down

e) CT scan

Young female was overweight and lost 30kg. Since then, she has been very selective about
eating and feels guilty after eating. When asked to step on the scale, she becomes teary and
says she takes laxatives to ‘clean her bowel’. She also says she will exercise after work. What
is the dx:
a. Body dysmorphic disorder
b. Anorexia Nervosa
c. Obsessive compulsive disorder
d. bulimia
e. She is fond of exercising

AfraTafreeh.com
Get more from: MplusX Qbank (https://member.mplusx.com/)
Facebook Page: (https://www.facebook.com/mplusxqbank/)

A 30 years old woman with increasing dyspnea, dry cough over few months with painful
lumpy skin lesions on legs and shins. On examination, there are red, painful, lumpy lesions in
leg and bilateral ankle swelling. ACE level increased. Which of the following is the best next
step to get diagnosis? (No given X-ray)
A. CT scan chest
B. Skin biopsy
C. Blood culture
D. Aspiration
E. ANA

A child presents with increasing fever and cough of 2 days duration. X-ray was given which
showed right sided pleural effusion. Which investigation aids in the definitive diagnosis of
the causative organism?
a. blood culture
b. chest x-ray
c. pleural aspirate
d. pneumococcal PCR
e. Sputum culture

35 year old lady comes to you regarding screening for breast cancer screening. She says her
paternal aunt was diagnosed with breast and ovarian cancer at the age of 60 years which was
diagnosed as being associated with BRCA1.Which of the following is the most appropriate
advice?
A-Genetic counselling assessment
B-refer for BRCA 1 screening
C-screen now
D-tell her to look at her breast
E-annual mammography

A mother came with son. 18years, h/o social withdrawal. Used to go to school and get good
grades. But stopped going including soccer practice, no hobby or social lift, watches TV at
night alone .dx?
A. schizophrenia
B. depression
C. schizoaffective disorder
D. bipolar disorder
C. adjustment disorder

A 9 yr old girl comes in with both parents come in to ask about the girl being pallor. Upon
examination, her hemoglobin count is 92 (105-115). You diagnosis it as B-thalassemia minor.
The parents ask what is the chances of their second child having this trait?
A. 0.0
B. 0.25

AfraTafreeh.com
Get more from: MplusX Qbank (https://member.mplusx.com/)
Facebook Page: (https://www.facebook.com/mplusxqbank/)

C. 0.33
D. 0.5
E. 0.75

Huge epigastric hernia picture of a 56 year old man which extends from umbilicus to
xiphisternum. He looked morbidly obese and had a waist circumference of 110 cm Asking
for appropriate management for the patient.
A. Weight loss therapy
B. Abdominal binder
C. Herniorraphy with mesh repair
D. Hernioplasty
E. Observation and Reassurance

16 yrs old girl comes in with bruises and cuts and the nurses approach her to get help her get
cleaned up. Patient becomes irritated and angry, yelling at you to stop helping her with her
injuries. You tell her it is important to get the wounds checked out, but patient still refused.
What is the appropriate step?
A. Take informed consent
B. Autonomy
C. Go ahead with the surgery
D. Seek hospital powers
E. Call police

An old man with onset pain in the lower back was given indomethacin. With little weight
loss, anal sphincter tone was normal. On DRE, prostate was bit enlarged and smooth. Patient
now complains of hematuria. What is dx?
A. Indomethacin induced hematuria
B. testicular cancer
C. abdominal cancer
D. acute pylonephritis
E. kidney failure

https://onlinelibrary.wiley.com/doi/pdf/10.1046/j.1464-410X.1997.02761.x

72yr old male presented with moderate confusion. On examination he is afebrile and nothing
abnormal detected. He had an ulcer on lower part of her leg treated by community nurse for
six months. (Pic given with ulceration and yellow pus). What will assist diagnosis
Urine test
Blood culture
Wound swab

AfraTafreeh.com
Get more from: MplusX Qbank (https://member.mplusx.com/)
Facebook Page: (https://www.facebook.com/mplusxqbank/)

CT scan
Lumbar puncture

A woman gave birth to a child on the 28th week. When examined, the child had small head
circumference hepato-splenomegaly and retinal pigments and visual defects. Most likely dx:
a. CMV
b. HIV
c. Rubella
d. Down Syndrome
e. Toxoplasmosis
Woman has been married for 45yrs and has only one sexual partner for 15yrs and that is her
husband. She comes to you complaining of malodorous green grey discharge from the
vagina. What is dx:
a) Bacterial vaginosis
b) Trichanella
c) Gonorrhea
d) Chlamydia
e) HIV

32yr woman with no hx of cancer or any illnesses, no f/h of cancer, comes for screening
advice?
a.3yrly fobt
b.2yrly colonoscopy
c.5yrly mammography
d.3 yrly pap smear
e. 2 yrly fobt (from 50 years)

55yr male with h/o surgery for ca esophagus; slight swelling in the lateral neck for3 weeks,
non-tender, smooth, soft. All examinations are normal. What’s your Dx?
a. branchial cyst
b. pleomorphic adenoma of parotid
c. submandibular duct stone
d. metastasis
e. oesophageal cancer

Pregnant lady delivered a baby and on 3th POD, she is worried the baby will be swapped so
should not allow the nurses to take the baby and sprinkles powder all over for protection.
Husband says the hospital environment is making the wife nervous so requests to take her
home.
a. Ask her husband to council her
b. Transfer her to shared room
c. Admit to ICU

AfraTafreeh.com
Get more from: MplusX Qbank (https://member.mplusx.com/)
Facebook Page: (https://www.facebook.com/mplusxqbank/)

d. Transfer her to psychiatric unit


e. Discharge upon request

Woman comes to you and says she will be travelling around Cambodia. What advice
regarding her travel to is most appropriate?
A Hep A
B.Hep B
C.Dengue
D.Malaria
E. Traveler’s diarrhea
https://wwwnc.cdc.gov/travel/destinations/traveler/none/cambodia

32-year-old nurse comes with relationship problems comes to the doctor with tears and
disclosure that she wants to kill herself. After a while, she recovers and admits that she is
stealing some blades from hospital and with some self-inflicted cut injuries in her hand. On
exam, you notice several superficial lacerations on her arm and wrists. what is most next
thing?
Report her to nurse supervisor
Give her anti-depressants
Tell her to go home for the day
Assess suicide thoughts
Inform to AHPRA

A child with hx of Strep. 10 days back was given Amoxicillin-Clavunate for Tx. Today, his
parents came back with him because he developed fever and tonsillitis; he also complains of
pain in wrist with skin rash present. what's the next step of management?
A. ESR
B. ABG
C. Throat swab
D. USG
E. CXR

A girl is taken to the hospital after being found unconscious. On her h/x, she had drank petrol
from a soft drink container, 500mg paracetamols/aspirin & 10 standard drinks. What is your
immediate tx?
A. give naloxone
B. give N-acetyle infusion
C. Take to surgery
D. ABG
E. gastric lavage

AfraTafreeh.com
Get more from: MplusX Qbank (https://member.mplusx.com/)
Facebook Page: (https://www.facebook.com/mplusxqbank/)

Patient wants to be your friend on social media what you should do?
Tell them they can discuss only medical related issues
They can message you directly
Accept the request but don’t contact directly
Decline the request
Delete your social media site

You are a working as a doctor and just withdraw blood from a patient who is suspected of
HIV. Unexpectedly, you stab yourself with the needle and the nurse sees it. What is your
management?
A. Wash hands with soap -next
B. Give patient reassurance you are fine
C. Administer gammaglobulin
D. Go to hemodialysis
E. Administer antiretroviral- best

A female patient comes to you for medical check-up. You notice she has bruises and cuts on
her limbs with a swollen left eye. She looks frightened and tells you that she does not want to
go home. What do you do next?
A. Advise her to go home
B. Call her husband and counsel him
C. Refer to domestic violence
D. Admit her to hospital
E. Tell her to call relatives and stay with them

A man was taken to you from prison after being aggressive and threatening to kill himself.
He is convicted and currently serving time for trying to kill his parents because he thinks they
have been replaced by double. Upon reviewing the h/x, you realize the man has been
diagnosed with schizophrenia as a child and given dexamethasone. The warden calls and asks
for your next step in management. What do you do?
A. Send the man back to prison
B. Administer anti-psychotics and send the man away
C. Release the man from custody
D. Send to prison hospital
E. Send to local psych unit

Mother comes to you saying that she recently had a divorce and noticed that her 8 year old
daughter has been acting differently. She states that she had found her daughter having
vaginal discharge and some bleed on her underwear. According, her daughter stays with her
but goes to her ex0husbands every other weekend. On P/E, there is no development of pubic
hair or breasts. What is ur dx?
a) Sexual abuse

AfraTafreeh.com
Get more from: MplusX Qbank (https://member.mplusx.com/)
Facebook Page: (https://www.facebook.com/mplusxqbank/)

b) Candidas
c) pre-puberty signs
d) Anti-social personality
e) adjustment disorder

Pregnant lady is sent to the ED after MVA in which the airbags failed to deploy. On P/E, she
is alert, conscious and responds to questions. Her vaginal exam did not have any bleeds and
CTG is normal with FHR 180/min. What is the next step?
A. Induce labour now
B. Inject synotocin to induce labour
C. Immediately C/S
D. C/S after 24hrs
E. Wait for labour

65 year old man undergoes surgery for ruptured abdominal aneurysm and has no other
problems. Upon examination, he shows no signs and reflexes are normal. What is your next
step in management?
A. Colloid solution
B. 3% saline
C. 0.9% solution
D. Transfuse pack of red blood cells
E. Oral iron
20-year-old is involved in an RTA with multiple f/x and undergoes surgery. 12hrs post-
surgery, patient is unresponsive but moans to the pain. What do you do next?
A. O2 via mask
B. O2 via prongs
C. Send back to surgery
D. Reassure
E. Nasal cannula
https://trauma.reach.vic.gov.au/sites/default/files/Early%20Trauma%20Care_Ver%20
1.0_25092014_complete_0.pdf

Old lady comes to you stating that she is feeling unwell. According to her h/x, she has
HTN, diabetes, hypothyrodism and cardiac disease for which she takes io-thyroxine,
metformin, verapamil, elondril? and thyroxine. Which tx will alleviate her discomfort?
A. Cease metformin
B. Cease verapamil
C. Cease elondril
D. Cease thyroxine
E. Increase thyroxine

Old lady comes in with SOB and dyspnea. She says she underwent surgery few months prior
and she has had asthma attacks and respiratory problems before however this is her worst

AfraTafreeh.com
Get more from: MplusX Qbank (https://member.mplusx.com/)
Facebook Page: (https://www.facebook.com/mplusxqbank/)

case. On exam, she is normal aside from a small reduced entry on the right side of the lung.
CXR shows small diffuse bilateral infiltrates. What is your dx?
A. Pneumothorax
B. Hemothorax
C. Pulmonary embolism
D. Fat embolism
E. Acute respiratory disease
A mother takes her 6 yr old son to visit you, complaining of hematuria with runny nose, skin
rash and fever. On exam, the child is responsive however feels unwell; during P/E, you notice
the legs have edema. The mother says he has been like this for few days. What do you
advise?
A. Take the boy home and restrict salt intake
B. Reassure the mother
C. Give the boy some medication
D. Tell the mother come back in several days
E. Admit the boy for fluid & salt restriction

A 26th week pregnant lady comes in to ask about gestation diabetes. She is worried that her
child will be diabetic and wants to know what to do. Her BMI is 26 and vitals are all normal.
She has a 3-year-old son. What would you advise?
A. Reduce BMI
B. Give metformin
C. Tell her that the child will be diabetic
D. Tell her that her child will not be diabetic
E. Avoid exercise

A man comes in complaining of gastric discomfort. His h/x states he drank lot of beers and
ate pizza the evening before. Then he felt discomfort and vomited in which the pain came
after. Vomitus contained just food particles and had no blood. On exam, the patient is
conscious but in discomfort and no jaundice. What is your dx?
A. Acute pancreatic cancer
B. Cirrhosis
C. Perforated duodenal ulcer
D. Alcohol liver disease
E. Mallory-Weise syndrome

A elderly man comes to the ED as result of status asthamaticus. He has been on albuterol and
salbutamol for years in which it has not alleviated the symptoms. What medication will you
give?
A. Prophylline + ipratrodium
B. B-agonist + albuterol + sedation
C. corticosteroids + salbutamol + sedation
D. Higher disease of albuterol

AfraTafreeh.com
Get more from: MplusX Qbank (https://member.mplusx.com/)
Facebook Page: (https://www.facebook.com/mplusxqbank/)

E. Corticosteriods only
https://emedicine.medscape.com/article/2129484-treatment#d10

A man comes to you complaining of paralysis on the right side of his face and arm. On
examining, you find everything is normal. What is your next step in tx?
A. Aspirin
B. Metoclomide
C. Dypyridamole/Aspirin
D. Morphine
E. Some nitrate

You are doing a research on chlamydia and specificity of the testing results. What is the
absolute result of people with chlamydia but testing negative. 2/27

Positive Negative
Infected 25 2
Non-infected ?? ??

72-year female come for routine checkup. Her mother developed colon cancer at the age of
65 years. She doesn’t have any bowel symptoms and tested negative on Pap smear three
times. She wants to know what other exams she needs to do. What do you recommend?
a) Reassure for no exams
b) Colposcopy every 3 yrs
c) Pap smear again
d) Vaginal exam every year
e) USG every 2yrs

A man went to Thailand for 5 days for his weekend getaway. Upon returning after two
weeks, he started to feet sick and had chills, fever, splenomegaly, and aches all over his body
for 2 days. He comes to you complaining of abdominal discomfort. On P/E, the liver is
palpable 3cm. What do you suspect?
A. Dengue
B. HIV
C. Malaria
D. Typhoid
E. E. Coli

A man is involved in RTA and is sent to the hospital. Upon arrival, the patient is clear,
conscious and alert. Patient does not complain of any pain and on exam, there is just bruising.
After three days, the patient complains of aches and is disoriented. What do you suspect?
A. Alcohol withdraw
B. Narcotic withdraw
C. Subdural hematoma

AfraTafreeh.com
Get more from: MplusX Qbank (https://member.mplusx.com/)
Facebook Page: (https://www.facebook.com/mplusxqbank/)

D. Pneumothorax
E. Lung contusion

Man brought to ER because he was acting in a bizarre manner at a shopping centre parking
area. His partner reports that he has been normal before this. In the ER the patient and severe
mood swings and agitation. He bends down, runs his fingers through the ground and says that
there are bugs running around on floor. Which of the following is the diagnosis?
a. Acute schizophrenia
b. Acute intoxication
c. Delusions
d. Drug abuse withdrawal
e. Schizophrenia

A party event was hosted where 20 people attended.


Food items Symptom Occurred Non-symptom No. of people
BBQ chicken 2 9 11
Cheese cake 10 2 12
Sandwich 10 8 18
Egg salad 7 10 17
Frozen berry smoothie 10 10 20
Which is the likely source of food poisoning?
A- BBQ chicken
B- Cheese cake
C- Sandwich
D- Egg salad
E- Frozen berry smoothie

28 year old male comes to visit you complaining of chest discomfort. On exam, you find
everything is normal and patient just says he feels faint. What is your tx?
A. Amiodarone
B. Adenosine
C. Carotid sinus massage
D. Defibrillate
E. Nitrates

74 year old man comes to you complaining of losing conscious. He has never had this
problem only when he finishes sexual intercourse. It usually occurs for few minutes when he
is at the bathroom. He says he does not have any problems after he returns to bed. What is
your dx?
A. Cardiac failure
B. Ischemic heart disease
C. Vasovagal syncope

AfraTafreeh.com
Get more from: MplusX Qbank (https://member.mplusx.com/)
Facebook Page: (https://www.facebook.com/mplusxqbank/)

D. Postural hypotension
E. TIA

A homosexual man comes to your clinic to pick up his medical records. You notice that he
has tested positive for HIV and you explain to him about his results. The patient agrees to
everything you said and tells you that he will use condoms but he will not tell his partners.
What do you do?
A. Listen to the patient and agree not to tell
B. Notify Health Department and start contact tracing
C. Tell him that it is your duty to inform his partners
D. Call police
E. Refer to infectious disease unit

A man is brought in for questioning his involvement for drunk-driving. He is oriented, alert
and conscious; his breath reeks of alcohol. Upon questioning, he says he has been having
problems and every night he would drink 4-6 glasses of beer. During lunch, he would have a
litre with his meal. When asked about alcoholism, the patient gets angry and says, “I am not
an alcoholic!” What is your most appropriate advice?
A. Tell him to cease drinking habits now
B. Explain about safe drinking levels
C. Refer to Alcoholics Anonymous
D. Call police
E. Thiamine
A 6 years old girl with history12 of severe sexual and physical abuse was moved into a
foster care. Her mother used to support her drug habit by working as a prostitute. The girl
although shy and quiet in the beginning, after a few weeks started showing sexual
behaviour with the neighbourhood kids. The foster mother wants to end the placement.
What’s your most appropriate management?
A. Do a general development assessment of the child
B. Organize a conference with child welfare services and the foster mother
C. Refer to child psychiatrist
D. Advice to end the placement
E. Call Child Protection Services

28-year-old female with history of multiple jail admissions and prostitution, and drug intake
comes to you for psychiatric evaluation from the court. She tells you that about how she left
school and home around 13 years old to start her life. On examination, you see that she is
quiet but anxious and when you ask her to tell more about herself, she becomes chaotic and
irritated. What is your diagnosis?
a) Antisocial
b) conduct disorder
c) borderline
d) drug abuse?

AfraTafreeh.com
Get more from: MplusX Qbank (https://member.mplusx.com/)
Facebook Page: (https://www.facebook.com/mplusxqbank/)

Huge eye pic: Patient presented with acute glaucoma what is the first thing you should give :
A. acetazolamide
B. timolol
C. pilocarpine
D. steroids
E. ice packs

A young girl is brought to you by her parents who explain that their daughter has been having
headaches, fever, chills, nausea and vomiting. Upon examination, you notice she is drowsy
and neck is stiff. What is your management in making a diagnosis?
A. ABG
B. CT scan
C. LP
D. Abdominal US
E. EKG
A young man comes to the ED after vomiting food particles. He says that he has never had
this problem before and it started when he was eating solid foods in which it became hard for
him to swallow. On exam, you notice there is narrowing ring of the esophagus. What is your
dx?
A. Esophageal cancer
B. Esophagitis
C. Peptic stricture
D. Esophagogastric cancer
E. Respiratory induced esophagitis

72-year-old male presented with mild confusion for several weeks. On examination he is
afebrile and nothing abnormal detected. He had an ulcer on lower part of her leg treated by
community nurse by several months. What will be the initial examination?
Urine test
Blood culture
Wound swab
CT scan
Lumbar puncture

Young male schizophrenic patient in psychiatric ward suffers ulcer in right arm, you were
informed by the nurse that the patient will not accept any intervention for this ulcer. What
will be your action?
A Ask consent from patient
B Seek more power in order to treat the patient

AfraTafreeh.com
Get more from: MplusX Qbank (https://member.mplusx.com/)
Facebook Page: (https://www.facebook.com/mplusxqbank/)

C Refer to surgical team


D Take a swab
E Neglect the ulcer

A 74 year old man comes to hospital with his family. His children tell you that he has been
acting weird and that during church sermon, he went up to the podium and talked about world
order. At home, he is sometimes irritated and anxious and sometimes he has tremors when
writing or feeding, but his memory is clear. What do you suspect?
A. Schizophrenia
B. Frontal lobe dementia
C. Paraphrenia
D. Drug abuse
E. Alzheimer’s

An old lady comes to you for advice that she has been feeling back pain. On exam, you
notice a small bony metastasis and tell her you will need to run some tests. Her h/x states she
undergone chemotherapy for gastric cancer. What is your next step in management?
A. Radiotherapy
B. Surgery
C. Chemotherapy
D. Supplements
E. No tx
Young man comes to visit you complaining of feeling abnormality in his testes. He says that
both testes feel normal however, there is a small protrusion on the upper side of his left
testicle and he can feel the vas deferens. On exam, the scrotum is normal but there is a
palpable mass on the left top. What do you suspect?
A. Teratoma
B. Hydrocele
C. Varicocele
D. Inguinal-testicle hernia
E. Testicle cancer

21year old come with left scrotal pain which wakes him up at night. No fever, malaise and
discharge. On examination, right scrotum and content is normal. Left scrotum tender,
redness, swollen. Cremasteric test is positive. Pain is reduced initially when left scrotum is
elevated. What is next step in management?
Urine C&S
Chlamydia PCR
Scrotal USG
Needle aspiration
CT scan

AfraTafreeh.com
Get more from: MplusX Qbank (https://member.mplusx.com/)
Facebook Page: (https://www.facebook.com/mplusxqbank/)

Mother brings her daughter to visit their local GP due to lesion on her body. On exam, the
doctor diagnosed it as impetigo and smeared cephalexin cream all over her body. Upon
finishing, her mother asks when can her daughter return to school?
A. After commencing cephalexin
B. After 2 days of antibiotics
C. After lesions have dried
D. After 7 days
E. After all lesions are gone

Man comes to psychiatrist for medical evaluation from court. His examination are all normal.
He has history of criminal record for assault, theft and larceny. After counselling section, he
confided to the doctor that he wants to kill his ex-partner. What is your next step?
a) Warn his ex-partner about it
b) Send him for forensic assessment
c) 3-4 continuous counselling session with him
d) invite his ex-partner for a session
e) call police

The patient has 8 months history of joint pain in the wrist and the ankle and minimal pain in
other joints but now the patient comes with morning stiffness of both wrists for 1 to 2 hours.
Her lab results as follow.
Hb - reduced
MCV - slightly reduced (nearly below lower margin level)
ESR - 72 (sure for this level)
A. NSAIDs
B. Prednisolone
C. Hydroxychloroquine
D. Methotrexate
E. Etarnarcept

Following a start of a dumping of waste by a factory, there is increase in cases of cancer in


the locality, what study will you do?
a) Cross-sectional
b) Case control
c) Cohort
d) RCT
e) Case report

Locals of one area affected by coal seam. Physical examination- WNL. What will be your
most appropriate action?
A. Inform public health unit
B. Educate about environmental risk

AfraTafreeh.com
Get more from: MplusX Qbank (https://member.mplusx.com/)
Facebook Page: (https://www.facebook.com/mplusxqbank/)

C. Inform public press


D. Reassure the people
E. Test the community

DM patient with injury to left index finger prior 2 days. Which indicate deep tendon
involvement?
A- Clear vesicle on finger tip
B- Pain on extensor tendon sheath
C- Flexion posture of finger
D- Pain on passive flexion of left index finger
E- Fusiform formation of finger

A man comes to you complaining of fever, headache, and rashes. You run a couple tests and
notice that he is positive for Coxiella Burnetti. Which of the following occupation is most
likely to be exposed to this virus?
A. Gardening
B. Working at a saw mill
C. Dairy farmer milking
D. Working at a reception desk
E. Sewage sanitation
Young girl comes to visit you because she has been feeling ill. She says she has runny nose,
fever, headache and dry coughs. On exam, the girl is alert but tired. Her mother asks when
can she return to school?
A. Immediately
B. After fever subsides
C. After finishing medications
D. After respiratory symptoms resolves
E. One week
Young man comes to you complaining of brisk epistaxis. He says he has never had any nasal
problems before. What do you suspect?
A. Digital trauma
B. Foreign objects
C. Sinusitis
D. Allergic reaction
E. Picking nose
Mother takes 9 mon old irritable baby with a bile stain vomiting and diarrhea to see her local
GP. She says the baby was doing fine until today when the baby kept vomiting up food.US
shows shadows in right upper quadrant. What next to do?
A. Urine culture
B. Stool culture
C. Air enema

AfraTafreeh.com
Get more from: MplusX Qbank (https://member.mplusx.com/)
Facebook Page: (https://www.facebook.com/mplusxqbank/)

D. Laparotomy
E. Laparoscopy

Pregnant women 16th weeks, oral glucose tolerance test 5.9. She is worried that she might
have diabetes. What is the next step?
a) Refer to dietician
b) start metformin
c) Follow up OGTT between 24-28 weeks
d) do hba1c every trimester
e) Reassure

A man comes to the ED after indulging several alcohol beverages. On exam, you notice there
is increased swelling on the abdomen with jaundice and abdominal distention. The patient is
in severe pain and you can palpate the liver. What is your diagnosis? (Pic is given)
A. Budd Chiari
B. Acute pancreatitis
C. Alcohol induced cirrhosis
D. Hepatomegaly
E. Splenomegaly

Woman comes to you complaining of fever, headache, rashes and a milky discharge from her
nipples for four months. Six months prior, she was prescribed estrogen-progesterone OCP for
birth control. What should you investigate?
A. FSH/LH
B. Serum ferritin
C. Serum prolactin
D. Testosterone
E. CA-125
26-year-old female, with hair on face and hands, with irregular periods 3-4 times a year.
What will you investigate?
A. Testosterone
B. Magnesium
C. FSH
D. LH
E. Vaginal US
A patient who was taking phenytoin for the past 10 years for epilepsy used to get 2 – 3
episodes every month. Several months back, he changed over to carbamazepine, after which
he did not have any more seizures for one month. Now he comes to you asking for
permission to drive. What should be the minimum interval before he can start driving?
1. 3 months
2. 6 months (I picked this cause he has been seizure free)
3. 12 months ???
4. 2 years

AfraTafreeh.com
Get more from: MplusX Qbank (https://member.mplusx.com/)
Facebook Page: (https://www.facebook.com/mplusxqbank/)

5. 5 years

(1)A middle age woman comes for Pap smear test. Her last 6 mth pap smear result is LSIL.
She also wants to take HPV vaccine, What is the app action?
A. Give vaccine now
B. Should not give vaccine
C. Vaccinate according to her Pap smear result
D. vaccinate according to HPV serology result
E. Vaccinate according to HPV PCR result

(2) Stroke patient with agitation, violent to staff, Most app action?
A. Send him home
B. Give sedation
C. Send to ICU
D. Send to separate room
https://academic.oup.com/ageing/article/36/6/613/40824

(3)
PCR + PCR -
Chlamydia test 26 1 27
+
Chlamydia test - 2 171 173
28 172 200
False negative result?
A. 2/172
B. 2/173
1.Study on Aspirin effects on preventing MI on 100 people. If those who took aspirin of the
100 ppl only 1 person had a MI. Of those who didn't take aspirin 2 ppl had MI. What is the
increase in relative risk in people not taking aspirin?
a. 1%
b. 10%
c. 100%
d. 200%
2. A 15year old girl having difficulty in concentrating. She constantly fights with father,
recently stopped ballet class – ask why “ what’s the point of all this ? “ ,what is the most
important in management?
A)Liase with father
B)Discuss the safety plan with her and her parents
C)Give SSRI
D)CBT
E)Tell school to support her more

2. A woman comes to your clinic. She was prescribed Trifluoperazine for her condition. She was
taking it for 3 years with improvement of her condition. She says that she discontinued
taking her medication for the last 3 weeks because her doctor was not present for he was in
trip, she also said that Trifluoperazine makes her hand or some muscle part stiff, rigid, and
restless. Now she presents with voices in her head. What is the most appropriate initial

AfraTafreeh.com
Get more from: MplusX Qbank (https://member.mplusx.com/)
Facebook Page: (https://www.facebook.com/mplusxqbank/)

choice in management?
A-Trifluoperazine
B-olanzapine
C-quetiapine
D-Respa depot
E-stop Trifluoperazine
3.
4. 12 yr old girl who is taken from her drug abuse parents and now living in fosters since she’s 2
year old , she likes toys and very proud of her big collection of toys, plays with her
immaginary friend , eat same foods always and very picky, her carer doesn’t like that , what
is the most immediate danger she is in ?
• 1- drug abuse
• 2-schizophreniform disorder
• 3-sexual abuse
• 4-anorexia nervosa
• 5-OCD
5.

5. Old lady keep losing track of time and forgetting things. when you examined her she
became agitated what is in her Mental Status Examination help you in diagnosis?
1 Constructional Apraxia
2 Orientation
3 Thought
4. Mood
5. Insight

6. Man 1 week after prostatectomy, his wife came to consultation because she is concerned
her husband is acting different, more irritated, aggressive, shouting. what in the history will
help u reach diagnosis?
A. Worsening symptoms at night
B. marked interpersonal aggression
C. history of cruelty towards animal
D. history of drug use
7. Young aboriginal male presents to you with insomnia, fear of darkness and seeing
“mamu”. He has been having these symptoms after the death of his mother. Which of the
following should be next step in his treatment?
A-give him benzodiazepine
B-consult to aboriginal health worker
C-urine drug screen
D-antipsychotic
E-drug and alcohol abuse counseling

8. Woman whose son is a drug addict, beats her every day. She is afraid and presents to you.
What will u do for the safety of the woman?
A. Inform her that you are obliged to inform the police
B. Go to the police
C. Tell her to go to refugee shelter

AfraTafreeh.com
Get more from: MplusX Qbank (https://member.mplusx.com/)
Facebook Page: (https://www.facebook.com/mplusxqbank/)

D. Advise her to go support group


E. Send the son to drug addiction service

9. 40 years old female brought by his friend in emergency,H/O Social phobia for 10yrs..She is
alone at home for many years due to social phobia.When at home,her interest in
Gardening,Reading,Now Which History describes her personality disorder for treatment??
A.School Refusal
B.Family H/O
C. Self-Harm
D. Alcohol abuse
No option for substance abuse

10. Scenario with female patient always angry, and can’t control herself, unstable
relationships. Self-harm was there, treatment was asked
a. anger management
b. dialectal behavioral therapy
c. Psychoanalytical therapy
d. Interpersonal therapy

11. Woman after death of her husband due to prostate cancer, lives alone ,6 weeks later
presented with loss of appetite, thinks he died because of her infidelity. She sleeps
badly(exact) and had similar episodes after the death of her child. Prompt treatment asked.
Temazepam
Venlafaxine
ECT
Mirtazapine
Escitalopram

12. A man was brought by his wife who had complained of sudden onset of tongue swelling. He is a
known diabetic treated with metformin and chlorpropramide, hypercholesterolemia given
simvastatin and hypertension for which he takes ramipril. His GP prescribed him amoxicillin 2 days
ago for a URTI. On examination, tongue is swollen. He has difficulty speaking and points to his
tongue. Wheezing can be heard on auscultation. Which of the following drugs is responsible for the
patient’s presentation?
A. Metformin
B. Chlorpropramide
C. Simvastatin
D. Amoxicillin
E. Ramipril

https://www.racgp.org.au/afp/2013/december/ace-inhibitor-angioedema/

13. Pt on Amisupride comes with gynaecomastia. Which drug may be the most appropriate next
drug?

AfraTafreeh.com
Get more from: MplusX Qbank (https://member.mplusx.com/)
Facebook Page: (https://www.facebook.com/mplusxqbank/)

a) aripiprazole
b) Clozapine
c) Olanzapine
d) Quetiapine

14. Couple come for infertility problem for the last 12 months. On testing examination and tests of
the female are unremarkable. Male has azospermia. And bilateral absence of vas deferens. Which of
the following is most important test before the starting the treatment of infertility?
A. No testing required as they cannot have a child
B. Testing of both male and female for cystic fibrosis
C. Refer for IVF
D. Serum FSH and LH for male
E. Serum Testosterone level

15.old patient who drinks about 10-12 standard drinks per day ,underwent some surgery.
on 3rd post op day he developed agitation ,O2 saturation 88%. what next ?
1-blood alcohol
2-blood glucose
3-CTPA
4-Chest X-ray
5-Urine RE
16. Poorly controlled Diabetic with mild renal impairment asking about risk for baby?
A. IUGR
B. Macrosomia
C. Renal agenesis
D. Intrauterine fetal demise
E. low Birth weight

17. A 44 years old man attends your OPD with complaint of increasing swelling over last 12
hours on his right thigh. He is a diabetic and hypertensive and has been previously operated
6 weeks back for a fracture femur. He is on Ramipril, Simvastatin, Warfarin, Amiodarone,
NSAIDs, and antacids. On examination you find that the right thigh is 4 cm greater in
circumference than the left thigh and is tender. His temperature is 37.8. Which of the
following is the reason for his presentation?
A. Arterial embolus
B. Cellulitis
C. Deep vein thrombosis
D. Edema
E. Drug interaction

18. 8 yrs old with type 1 diabetes on insulin glargine. Every morning glucose levels is high.
What to do ?
a. check blood glucose levels at 2:00-3:00 am for 3 days
b. check early morning insulin levels
c. increase the evening intermediate insulin
d. give insulin before breakfast
e. give another dose before sleep

AfraTafreeh.com
Get more from: MplusX Qbank (https://member.mplusx.com/)
Facebook Page: (https://www.facebook.com/mplusxqbank/)

19. Young woman presents with epistaxis,fatigue,tiredness for months,menorrhagia.she is


pale ,not in distress, On investigation Hb-85g/l,MCV-low,Ca-1.9,INR-1.5(normal 1.1)SURE.
Which of the following is appropriate next step ?
A-blood transfusion
B-IV Calcium
C-Oral Iron
D-fresh frozen plasma
E- Vit K injection
Exact options

20. Young man with rectal bleeding and found 10cm friable mass on sigmoidoscopy.
management?
a.rectalsteroid
b. sulphasalazine
c. methotrexate

21. Peptic ulcer case with Hpylori positive, took triple therapy (Amoxil+metro+PPI) for a
week or 10 days (not sure). Urea breathe test still positive after 6 weeks. Asking the reason?
- Resistant to metronidazole
- Resistant to amoxil
- Not reliable urea breath test
- review in next week

22. Female patient with RA she takes ibuprofen and methotrexate to control her disease for
5 years. Recently patient complains of some symptoms (jaundice), and her labs are given.
protein - 9 (6-8) slight increased
albumin- 5 (3.5-5.5) normal
ALT - >100 (7-56)
AST - >1500 (10-40)
GGT - >100 (0-30)
Liver enzymes are >100

What is the cause of her symptoms?


a. Methotrexate induced hepatitis
b. ibuprofen induced hepatitis
c. Autoimmune hepatitis
d. Viral hepatitis

23. A mother came with her two children. Mother's brother has haemochromatosis. What screening
test advice would you offer?

a) Screen only mother


b) Screen mother and kids
c) Ask her to come with her husband
d) Screen only the children

AfraTafreeh.com
Get more from: MplusX Qbank (https://member.mplusx.com/)
Facebook Page: (https://www.facebook.com/mplusxqbank/)

24. Child presented with UTI, on USG the right kidney is smaller than the left. What is the best
investigation to assess renal function?

a) DMSA

b) DTPA

c) Urine culture

d) CT scan

25. 38 years old man with renal transplant 9months (exact) back, presented with Neck
stiffness and meningitis sign start 3 days ago, nothing mentioned about respiratory
symptoms, no fever. On chest xray there is well defined round opacity in “middle right lung”
( chest X-ray not given, just mentioned secenario). What is the cause?

A. Aspergillous
B. TB
C. Pneumocystis Pneumonia
D. Lymphoma
E. Nocardiasis

26. Child with previous URTI,RBS +, Protein + in the urine dipstick test. After two weeks
urine shows only RBC + and from non-glomerular origin. What is the appropriate
investigation?
A. Urine cytology
B. Bladder scan
C. Renal ultrasound
D. Urine culture
E. Serum electrolytes and creatinine

28. Patient with rheumatoid arthritis and chronic pyelonephritis for years. On methotrexate
well controlled. Also become anemic lately, with microcytuc anaemia. What is the
appropriate management for this patient?
A. Packed RBCs
B. IV iron
C. Low dose corticosteroid
D. Erythropoietin stimulating agent
E. Folic acid

29. Patient comes with morning stiffness of both wrist for 1-2 hour. She has this history for 8 year.
Lab result shows HB - reduced, MCV - slightly reduced, ESR - 70. Next Rx?
A. NSAID
B. Prednisolone
C. Hydroxychloroquine

AfraTafreeh.com
Get more from: MplusX Qbank (https://member.mplusx.com/)
Facebook Page: (https://www.facebook.com/mplusxqbank/)

D. Methotrexate
E. Etarnarcept

30. A man comes with clumsiness of hands and tripping over. RightUpper limb weakness
and fasciculation.Left Lower limb muscles also has weakness, increased tendon reflexes.
Planter response is equivocal. No wasting. No sensory loss is given. What is the initial
investigation?
A. EMG
B. MRI Spine
C. CT scan
D. CSF examination
E. Cervical spine X-ray
31. Patient with parkinsonism like features , history of taking resperidone and now visual
hallucination. His cognitive function is impaired for 6 month. What is the most likely
diagnosis?
a. Dementia
b. Lewy body dementia
c. Drug interaction

d. Cerebral infarct
e. Parkinsonism
32. Middle age man present with retrosternal chest pain. This is preceded by vomiting. On
examination, reduced breath sounds in lower lobes of lungs and dullness over these. Next
appropriate investigation?
a. CXR
b. Gastrograffin swallow
c. CTPA
d. Non contrast CT chest
e. ECG

33. A lady with BMI of 35. How will you manage her in addition to exercise for long term
management?
A. Low Carbohydrate food
B. Lipase inhibitor
C. Diuretics
D. Surgery
E. 4000 kJ/ day= 1000 kcal

34. 65 year old man presented with loin pain, urine examination showed hematuria. What is
the most appropriate next step?
A-X-ray
B-abdominal ultrasound
C-CT abdomen

AfraTafreeh.com
Get more from: MplusX Qbank (https://member.mplusx.com/)
Facebook Page: (https://www.facebook.com/mplusxqbank/)

D-MRI
E-urine culture

35. A women on multiple drugs one of them was digoxin, she presented with central
abdominal pain and tenderness. Her pulse was irregular. Apart from a contrast abdominal ct
what is appropriate next investigation?
A. Abdomen USG
B. Serum Lipase
C. Serum lactate
D. Digoxin level
E. INR

36. A patient with a painless neck swelling moves with swallowing. Now hoarseness and
difficulty in breathing at night. increased over 3 weeks. O/E smooth swelling in the anterior
triangle of neck. dullness of percussion at upper sternal border. most likely dx? Nearly like
this scenario

a. Anaplastic cancer
b. papillary carcinoma
c. MN Goiter
d. hemorrhage in nodule of MND
e. follicular CA

37. Similar picture, but rib # is not prominent like this


Patient is in severe pain and cannot breathe properly. What is the cause of pain (Exact
words)? (A long scenario)
A. Fractured rib
B. Haemothorax
C. Pneumothorax
D. Haemopneumothorax
E. Tension pneumothorax

AfraTafreeh.com
Get more from: MplusX Qbank (https://member.mplusx.com/)
Facebook Page: (https://www.facebook.com/mplusxqbank/)

38. 55 year old man, history of appendicectomy and cholecystectomy presented with 3 wk
history of abdominal pain, distension with tympanic bowel. No vomiting only mild
distension. X-ray shows dilated ascending colon, transverse colon and descending colon, no
air. Rectum is empty and normal looking rectum. No picture X-ray
a. Sigmoid volvulus
b. Adhesive IO
c. CA Sigmoid colon
d. Cecal Volvulus
e. Intussusception

not exactly, apple core in middle


39. 65 year old man, History of appendicetomy& cholecystectomy presented with 3 wk
history of abd pain, BS exaggerated no air in rectum likely diagnosis?
a. Sigmoid volvulus
b. Adhesive IO
c. CA Sigmoid
d. Pseudo obstruction
e. Intussusception

40. A young man complaint, he noticed a swelling in his groin after lifting
weight yesterday, on examination there was 1cm defect in inguinal area with fat protruding
in the inguinal canal. On ultrasound, the defected is protruded from deep inguinal
ring. What will be the next appropriate treatment?
A. Reassurance
B. Open mesh repair
C. Laparoscopic repair

41. A man presents with c/o left leg pain, can walk up to 100 meters , due to pain has to
rest for sometimes relieved by rest, on examinations there was , right leg good peripheral
pulses, left leg weak pulses , ABI done and it was 0.25(exact value). What is the most
appropriate test leading you to the diagnosis?
A. Arteriography
B. CT angiography

AfraTafreeh.com
Get more from: MplusX Qbank (https://member.mplusx.com/)
Facebook Page: (https://www.facebook.com/mplusxqbank/)

C. Compression Doppler ultrasound


D. MRI
E. X-ray

42. An old man with acute onset very severe pain in the lower back with some urinary
symptoms (think it was retention).significant weight loss. Anal sphincter tone was normal.
On DRE enlarged irregular prostate. What next. Dullness on palpation 2 finger breaths above
symphysis pubis.Slightly elevated temperature.
A. Chest x-ray
B. CT spine
C. TRUS
D. Bladder scan
E. PSA
43. 65 yr old man with nocturia and long hx of dribbling urine problems with median sulcus
palpable ,smooth prostate what next
A. Urine cytology
B. PSA
C. Bladder scan
D. CT scan abdomen
E. IVU

44. 60 years old man is having a check-up for prostate cancer last two years PSA was
1.5mg/ml, now PSA is 3.8mg/ml, then 12 slides of biopsy done and only one foci showed
adenocarcinoma. Gleasonscore is 4. How will you manage this patient?
A.TURP
B. Radical prostatectomy
C. Active surveillance
D. EBRT
E. Androgen therapy
45. 45 yrs old man with no family history of prostate cancer comes for screening advice and
counselling . He insist he wants a DRE.A digital rectal exam showed a normal sized prostate
with normal non nodular sulci. What is the next important step?
A. Counsel him and return if any symptom
B. PSA
C .TRUS
D. Abdominal usg
E. Genetic counselling

46. A patient of CKD and hypertensive, patient taking ramipril, carvedilol with many other
drugs presents with confusion labs
urea is 18
creatinine 520

AfraTafreeh.com
Get more from: MplusX Qbank (https://member.mplusx.com/)
Facebook Page: (https://www.facebook.com/mplusxqbank/)

raised calcium 3.1


phosphate also raised
sodium 128
potassium 5.5
Exact value
What is the cause of confusion in this patient?
A. Uremia
B. Hyponatremia
C. Hyperkalemia
D. Hypercalcemia
E. Hyperphosphatemia

47. 58 year old lady with DEXA -2.5 for vertebrae and femur, with low vitamin D levels and
normal calcium, past history of GERD. What is the appropriate management for her?
A. Strontium ranelate
B. Calcium &Vit D
C. Alendronate
D. HRT
E. Calcitonin

48. Mother living in mining town comes with her daughter for normal blood test. Her daughter’s
test show lead level 0.75 (normal <0.35) what will be your next initial step?
Exact value of lead level
a) Educate mother about environmental risks
b) Assess IQ testing of child
c) Refer for chelation therapy
d) Move the family to another town
e) Urine test to confirm dx

50. Few days old infant presented to you complained with poor feeding and bile stain
vomiting. His Birth was at 38 weeks gestation and weight 2600 g. he is breast feeding with
no immediate post-natal complications. The child has a history of passing meconium on day
4 of birth. Now He has mild jaundice and abdominal distension, and is irritable .On
examination, abdomen distended and tender. What is the appropriate diagnosis?
A. Meconium ileus
B. Hirschsprung’s disease
C. Duodenal atresia
D. Necrotizing enterocolitis
E. Volvulus
51. 8yr old child brought by his mother , he complained from episode of staring suddenly that occur
along with fidgeting of right hand and movement of right arm head twitching to right side,
sometimes chewing and swallowing each episode last for 60-90 second then the child remain dizzy
and confused for 1-2 minutes after the episode . These occurs in 3-4 days and then the child back to
his normal activity and behaviour for several weeks .what is the most likely diagnosis?
Nearly exact scenario
A- Temporal lobe epilepsy

AfraTafreeh.com
Get more from: MplusX Qbank (https://member.mplusx.com/)
Facebook Page: (https://www.facebook.com/mplusxqbank/)

B- Juvenile myoclonic epilepsy


C- Absence seizure
D-Myoclonic seizure
E. Tourette

52. A mother came with her child complaining of continuous bleeding after fall from coffee table. On
exam there is MINIMAL bleeding from the frenulum and child had some old bruises on forehead and
legs. there was no petechiae, lymphadenopathy and hepatomegaly. What is the most likely cause?
a. Idiopathic Thrombocytopenic Purpura
b. Non accidental injury
c. VWD
d. clotting defect

53. A girl want go to India low budget trip she has had previos H/o oral polio, 1 MMR in
childhood ,dpta 10 years ago. apart from typhoid and influenza what other vaccines bud u
give?
a. MMR
b. mmr polio
c. Polio, mmr, dtpa
d. mmr, dtpa
54. Child with fever 39 C and cough, Chest x-ray (Lt lower lobe consolidation) given. What is
the most appropriate next step investigation?
A. CT
B. Blood culture
C. Pneumococcal pcr

D. Lumbar puncture
E. FBC
55. 6month old child came for check-up. On questioning about vaccination, mother said she
had only 2 (1 at birth and 2nd on 2months old) child is having recurrent URTI. On exam,
child is alert, T- 37.8 c, clear discharge from nose. What is the next management in terms of
her vaccination?
a. Investigate underlying cause of urti
b. Hbs vaccination now
c. Vaccine after fever subsides
d. Hbs vaccination now and register for catch up vaccination schedule
e. Register/give as per vaccination schedule
56. A child with fever and hx of sore throat. He developed pain in wrist later developed
arthralgia n swelling in ankle joint. skin rash present on back . What's the most initial step of
management?No Murmur mentioned.
A. ESR

AfraTafreeh.com
Get more from: MplusX Qbank (https://member.mplusx.com/)
Facebook Page: (https://www.facebook.com/mplusxqbank/)

B. Full blood examination


C. throat swab
D. USG
E. ECG
57. Lady full term with adequate contraction was monitored on CTG. Everything was
progessing normally. Suddenly there was a deceleration of FHR dropping to 70bpm for 4
mins. What is the next most appropriate step?
a. Stop syntocinon
b. Continue synto at an increased dose
c. Immediatecessarean
d. Ventouse delivery

58. 39 weeks pregnant lady came with labour pain. Synto was given appropriately. Ctg was
done which shows heart rate of 140 which dropped to 70 and came back to 140 in 2min.
asking next appropriate treatment. (Que does not include any basic measure given for her
like Left
lateral position, Oxygen, Fluid)
A. Stop syntocinon
B. Fetal scalp sampling
C. C section
D. Titrate to increase dose of syntocinon
E. Give O2 to mother via face mask

59. A 25 years old primigravida presents for results of antenatal screening tests. She is a
drug injector in the past. But has not get any drugs in the last 5 years. Her results show Hep
C Ab +ve, Hep C RNA –ve, HIV –ve, Hep B Ab +ve. Which of the following will reduce the risk
of transmission of Hep C to her baby?
A. LSCS at term
B. Formula feeding of newborn
C. Avoid pH monitoring of fetal scalp electrodes
D. Anti-retro viral therapy

60.A 45 yr old lady smoker and htn uses ocps for many years, BP 150/90mmHg now comes
re prescription because she want to controlled her menorrhagia and also says doesn't want
to concieve. What to do now?
Nearly exact scenario
A. Tell her stop smoking then can give ocps
B. Use implanon
C. Use copper iud
D. Use progestrogen only pill
E. Use condoms +ocps

61. Young woman 26 years of age comes to you complaining of heavy menstrual bleeding
with pain. She describes that clots of blood are lost in the first few days of menses with
severe back pain. You ask her for some investigations that need to be performed. She gets
irriated and says she just wants a treatment. Which of the following is the apporpraite next

AfraTafreeh.com
Get more from: MplusX Qbank (https://member.mplusx.com/)
Facebook Page: (https://www.facebook.com/mplusxqbank/)

treatment ?
a. Tranexemic acid from day 1 of menses
b. Mefanimic acid from day 1 of menses
c. Mirena
d. Oral contraceptive pills

62. 48 year old woman complains of heavy menstrual bleeding for last 4 months. She underwent a
hysteroscopy and D&C for 3 month ago but the symptoms did not subside. She looks pale and Her
haemoglobin is 8.5 g/dl(exact value). What is the most appropriate management in this case?
A-oral tranexamic acid during the period
B-northisterone from 15-25 days of cycle
C-mirena
D-implanon
E-COCP

63. A 23 year old lady presents to you asking for advice regarding the need for Chlamydia screening.
Her last year pap smear was normal but now she wants to check for STD screening.
Reason for chlamydia screening

A. It can be asymptomatic.
B. It causes infertility.
C. Her partner could be asymptomatic.
D. She does not need the test

64. A indigenous boy had sex with new sex partner. After a few days he found urethral discharge
from his penis. What specimen will help you make a diagnosis of the STI?

Nearly exact scenario


a.1st catch urine chlamydial PCR
b. Mid-stream urine chlamydial PCR
c. urine culture
d. Urethral swab gram stain and culture

65. Woman 51 years of age presents after menopause for the last 3 years. She complains of
irritability, moodiness, low libido and hot flushes. Which of the following would be most
appropriate to give to this woman?
a. Combined continuous estrogen and progesterone throughout the cycle
b. Continuous estrogen throughout the cycle
c. Continuous estrogen with progesterone first 12 days of the cycle
d. Continuous progesterone with testosterone

66. Woman with a history of hysterectomy and DVT. After trying non-hormonal therapy, she still has
complaint of hot flushes and insomnia. What will you give her?
A. Oral low dose estradiol
B. Transdermal estradiol
C. Progesterone only pills
D. HRT
E. Clonidine

AfraTafreeh.com
Get more from: MplusX Qbank (https://member.mplusx.com/)
Facebook Page: (https://www.facebook.com/mplusxqbank/)

67. lady comes on 10th postpartum day with pain due to perineal tear. Now on examination there is
perineal wound that is 2 cm long,1cm wide and 1 mm deep.It was clean with no discharge.
What to do next?
Apply local antibiotics
Put on oral antibiotics
Simply keep the wound clean
Suture the wound under local anaesthesia
Suture the wound after cutting the edges

68. 15 wks pregnant comes for risk of Down syndrome. What is the diagnostic test for her(Exact)?
a) amniocentesis
b) CVS
c) nuchal fold thickness
d) serum screening

69. Insufficient breast milk production, reason?


a) insufficient fluid intake
b) not putting for breastfeeding frequently
c) not enough time latching during feeding
d) exhaustion
e) poor positioning

71. Young couple with infertility. Female with one child in previous marriage and female
investigations were normal, what in history you will ask the male patient that will be most
useful to guide u to cause of infertility?
A. Do u drink alcohol so much?
B. Did u shave often?
C. Do you use Marijuana?
D. Do u have chicken pox?
E. Do u often use ectasy and party drugs?

74. Young man after a quarrel had a fracture of floor of eye what is the most consistent symptom
with that?
a-Conjunctivalhaemorrhage
b-loss of visual activity
c-anaesthesia around the cheek
d-Cant open the mouth completely
e- Epistaxis

75. Indigenous child media presents with chronic ear discharge. Ear swab done showed existence of
pseudomonas aeruginosa organisms. After ear toileting what will u do?
A. Oral Amoxicillin
B. Oral Augmentin
C. Ciprofloxacin ear drop
D. Paramycetin with steroid ear drops
E. Hearing aids

https://emedicine.medscape.com/article/226748-treatment#d5

AfraTafreeh.com
Get more from: MplusX Qbank (https://member.mplusx.com/)
Facebook Page: (https://www.facebook.com/mplusxqbank/)

76. What is the most appropriate management? Exact picture

A. Local excision
B. Excision with a 2 cm margin.
C. Fluorouracil cream
D Review in 12 months

77. 40 years old lady complaining of back pain after gardening On examination, there was
tenderness in the lumbar vertebrae L3 / L4 with limited flexion and extension. After giving analgesics
what is your next line of management?
A. X-ray
B. observation
C. refer to orthopedics
D. corset
E. MRI

78. 65 years old lady complaining of back pain after gardening On examination, there was
tenderness in the lumbar vertebrae L3 / L4 with limited flexion and extension. After giving
analgesics what is your next line of management?
A. Bed rest
B. continue activity
C. refer to orthopedics
D. corset
E. MRI
(No XR in option)

79. what is the best measurement for health risk in obesity?


a. BMI
b. Waist circumference
c. Waist hip ratio
d) Body weight
e) Fat distribution

AfraTafreeh.com
Get more from: MplusX Qbank (https://member.mplusx.com/)
Facebook Page: (https://www.facebook.com/mplusxqbank/)

80. 75 yr old man living alone found to have increasing forgetfulness. Diagnosis?Exact
picture and options
A. cerebral infarct
B. cerebral tumor
C. cerebral abscess
D. cerebral hemorrhage
E. subdural haematoma
81. A 48 years man presents with symptoms of claudication. His BP IS 140/80 and he has 40
pack year smoking history. You assess him and decide he does not require intervention. He
asks you what is the most important step in reducing his symptoms.
A. Reduce smoking (not stop, reduce)
B. Reduce BP
C. Supervised exercise
D. Atorvastatin

82. He sustained wound on that arm 1 month back but it was healed. Drinks 1 glass of wine
every day and smokes 10 cigarettes a day. Which of the following is the most common
cause?
A. Farm worker
B. Chronic infection
C. Genetic
D. Smoker
E. Alcoholism

83. woman works at a part-time job, and complaints of headache throughout the day, frontal &
bilateral, dull in character, varying in intensity, she takes paracetamol& ibuprofen, which only cause
relief for 2-3 hours. What is the cause?

A. Drug rebound headache*

B. Migraine

AfraTafreeh.com
Get more from: MplusX Qbank (https://member.mplusx.com/)
Facebook Page: (https://www.facebook.com/mplusxqbank/)

C. Tension headache

D. Cerebral tumor

E. Pre-menstrual headache

84. post op oliguria after cholecystectomy. UO 100 ml /12 hrs .catheterised. fever present 38.9.

A. Serum Cr
B. Bladder scan

C. blood culture
D. abdominal USG

E. abdominal X-ray

85. post op pt oliguria but no fever and no mention of the catheter , what is ur most appropriate
management?
A. abd x ray
B. blood culture
C. serum electrolytes and Cr
D. bladder us

86. long history about 55 yr postmenopausal female pt concerned about risk of osteoporosis , she
had done an oesohageal operation and still complaining of currentsome GiT problems(sure), her t
score cevical 2.5 and Lumbar T score -2.5, they give lab their ca level is normal and vit d level is
normal and no mention in the Q about any menopausal symptoms,what is your most appropriate
management ?
A. alendronate
B. ca plus vit d
C. HRT

D. Strontium ranelate

87. A man comes with clumsiness of hands and tripping over. Upper limb weakness and
fasciculation.Lower limb muscles also has weakness, increased tendon reflexes. Ankle reflex is not
increased in one side. No wasting. No sensory loss is given. What is the initial investigation?

Nearly exact scenario


A. EMG
B. MRI Spine
C. CT scan
D. CSF examination
E. Cervical spine X-ray

88. An old lady lives by herself mmse 28 had fracture femur. Family wants everything to be done she
also has copd and has to be admitted icu many times a day. This is her 6th time .Doesn’t want
further treatment .. Lady is competent to make her decisions
A .Psychoneurologicalassessemnet
B. Arrange meeting with the family
C . Inform the hospital ethic group

AfraTafreeh.com
Get more from: MplusX Qbank (https://member.mplusx.com/)
Facebook Page: (https://www.facebook.com/mplusxqbank/)

D. Respect her wish

90. Mother of 2 pre-school children come to consult d/t inter-relationship conflict. She
request her partner to leave the room and said “I am really wanna give up on taking care
of my children”. Which is next?
A- Call the police
B- tell her partner also need to be informed
C- for psychiatric admission
D- inform child protective care centers
E- Explore more about her words

92. A 44 years old man attends your OPD with complaint of increasing swelling over last 12
hours on his right thigh. He is a diabetic and hypertensive and has been previously operated
6 weeks back for a fracture femur. He is on Ramipril, Simvastatin, Warfarin, Amiodarone,
NSAIDs, and antacids. On examination you find that the right thigh is 4 cm greater in
circumference than the left thigh and is tender. His temperature is 37.8. Which of the
following is the reason for his presentation?
A. Arterial embolus
B. Cellulitis
C. Deep vein thrombosis
D. Edema
E. Drug interaction

93. 21years old come with left scrotal pain which wakes him up at night. No fever, malaise
and discharge. On examination, right scrotum and content is normal. Left scrotum tender,
redness, swollen. Cremasteric test is positive (cannot be torsion). Pain is reduced initially
when left scrotum is elevated. What organism is mostly associated?
A- E coli
B- Ganorrhoea
C- Chlamydia trachomatic
D- Sypilis

94. Ethiopian migrants presents for medical check up, Mantoux test 12 mm, history of BGG
vaccination, HIV negative. Next app step?
A. CXR
B. repeat Mantoux
C. Observation
D. INH given
E. Isolation
https://www.ncbi.nlm.nih.gov/pmc/articles/PMC3481914/

95. Man with abdominal pain, Calcium increased, history of progressive dyspnoea for 3 year.

AfraTafreeh.com
Get more from: MplusX Qbank (https://member.mplusx.com/)
Facebook Page: (https://www.facebook.com/mplusxqbank/)

What inv?

A. parathyroid hormone
B. serum ACE
C. Anti GBM

98. Drug efficacy and safety?


A- RCT
B- Cohort
C- Case-control
D- Cross-sectional
Systemic review

100. Women taking trifluoperazine (1st generation antipsychotic) feels like volcanic eruption
in her stomach lost contact with psych.stop drug dt movement disorders asking Rx
a. triflouperazine
b. haloperidol
c. quetapine
d. clozapine

101. Pt on Amisupride comes with gynaecomastia. Which drug may be the most appropriate next
drug?

a) aripiprazole
b) Clozapine
c) Olanzapine
d) Quetiapine

102. Sarcoidosis case. Lymphadenopathy, rash (erythema nodosum), conjunctivitis, and


bleeding. Cough +. Skin nodules +. ACE increase. What invx??

Skin biopsy

103. 14 years old girl lives with family has now become sexually active comes to you for
contraception advice and tells you not to inform her parents. What will u do?
A. Give her prescription of contraception
B. Ask her about identity and age of her sex partner
C. Tell her she needs parents’ permission
D. Inform parents
E. Tell her to use condom to her partner

104. Restless leg syndrome scenario. What is the next investigation?

AfraTafreeh.com
Get more from: MplusX Qbank (https://member.mplusx.com/)
Facebook Page: (https://www.facebook.com/mplusxqbank/)

a. Sleep studies
b. Nerve conduction test
c. Iron studies
105. A patient with spiking fever, SOB and was inserted prosthetic heart valves 3 months
ago. And 3 blood cultures done dx as due to staph aureus IE, and given flucloxacillin for
treatment. The fever subsides after 2 weeks of treatment, but SOB still persists,
diastolic murmur at the left sternal edge? What will u do for investigation?
a) CXR
b) troponin
c) blood culture for every week
d) trans thoracic ECHO

106. year old retired farmer presented with left groin lymph node enlargement. The biopsy
specimen shows squamous cell carcinoma. what is the primary site?
a. anus
b. rectum
c. prostate
d. left leg
107. A woman came for interview asked about appetite she said one doesn't get cheese
from moon and I came by bus What is diagnosis
Disorganized behavior
Depression
Delusion
Depersonalization
Dissociation

108. You are intern. The patient is intubated and give sedation. You are called to his bedside
and wanted to be a witness to change his will. What should you do?
Witness the will with lawyer
Refuse to witness
Ask surgical register if he can do it
Seek legal advice
Ask the nearby kins and witness the will

109. 24 months child verbal milestone?


2 pronouns

110.A man is brought by policemen after beaten his friend during an argument, he is
intoxicated for alcohol what in history will be important to say personality disorder
History of fight with colleague two weeks ago
History of cruelty to animals in childhood
Alcohol and drug abuse
111. Girl 18 years history of weight loss BMI 15. Her school performance is good and
recently fall in grade. When asked about any interests said why should I? What
should that entire world mean? Which of the following will have in the history of
her?
A. Episodic fatigue (exact words)

AfraTafreeh.com
Get more from: MplusX Qbank (https://member.mplusx.com/)
Facebook Page: (https://www.facebook.com/mplusxqbank/)

B. Sleep disturbances
C. Suicidal idea***
D. Feeling inferiority*
E. idea of guilt

112. A woman, no family history of DM, for screening

A- FBS 3 yrly

114. 92-old patient found unconscious at home brought by son to emergency


department. the patient was taking tablets methadone, oxycodone , others , patient
has constricted pupil on examination which of the followings is the likely cause?
a. methadone
b. oxycodone
c. other options not remember
115. A child 6 years old with history of asthma presented with upper respiratory
tract infection, urine examination showed very high serum glucose and ketone body
of 2+. What is the most appropriate test to follow up this child?
A. HbA1C
B. Serum creatinine and electrolyte (if DKA)
C. FBS
D. OGTT
E. ABGA

1.2 yrs old chid comes from bengladash present with cough for past month . no fever . on
chest X ray ,showing right hilar lymph adenopathy and lower lobe infiltrate (just mention
not given )asking next investigation.
A.mantox test
B.nasopharyngeal aspirate
C.bronchoscopy

2. A university student present for sick certificate that he missed exam yesterday due to
muscle aches,headaches and fever starting last three days and he is now well . what will you
do ?
A.Decline the request as his illness is not confirmed
B. Decline the request as he is not present during the exam day
C.confrim his illness from third party
D.Dating the certificate with today date
E. Dating the certificate with the exam date

3.72 years old female comes for screening. Her mother is diagnosed with Ca colon at the
age of 68 years old and she is otherwise well . What will you do for her?

AfraTafreeh.com
Get more from: MplusX Qbank (https://member.mplusx.com/)
Facebook Page: (https://www.facebook.com/mplusxqbank/)

A.No further screening test is necessary


B.Fobt 2 yearly
C.clonoscopy
D.sigmoidscopy

4.54 years old man comes for colon cancer screening after seeing National colon cancer
screening on TV.What will you advice ?
A.fecal occult blood test
B.colonscopy
C.FBC

5.A patient call GP concerned about her 19 years old daughter .The daughter consulted with
you and you suspected that she is using ectasy and you prescribed the sleeping pill.What
will you do?
A.tell that you prescribed the sleeping pill
B.Tell that you cannot disclose about her daughter
C.to come with her daughter together in next appointment

6.72 years come with colicky central abdominal pain then pain becomes constant .On
abdominal examination , abdomen is soft ,no gurding and tenderness at the LIF only.BP-
13/85 .PR-100/min.
Appendicetomy done at last 17 years ago.
A. mesenteric adenitis
B.diverticulitis
C.small bowel obstruction

7. 64 years old woman complaint of generalized malaise ass: with progressing fatigue
and persistent thirst for some duration. Her blood chemical results are:
Na+ (normal), K+ (normal), HCO3- (normal), Chloride (normal),
Total Ca+ - 2.65 (2.1 - 2.55)
Ionized Ca+ 1.38 (1.1 – 1.35)
What invx?
A. 24hrs urinary calcium
B. Glucose tolerance test
C. Serum parathyroid hormone assay

D. Bone scan
E. Serum angiotensin converting enzyme

8. DM patient with injury to left index finger prior 2 days. Which indicate deep tendon
involvement?
A. Clear vesicle on finger tip

B. Pain on extensor tendon sheath

AfraTafreeh.com
Get more from: MplusX Qbank (https://member.mplusx.com/)
Facebook Page: (https://www.facebook.com/mplusxqbank/)

C. Flexion posture of finger

D. Pain on passive flexion of left index finger

E. Fusiform formation of finger

9. A 44 years old man attends your OPD with complaint of increasing swelling over last
12 hours on his right thigh. He is a diabetic and hypertensive and has been previously
operated 6 weeks back for a fracture femur. He is on Ramipril, Simvastatin, Warfarin,
Amiodarone, NSAIDs, and antacids. On examination you find that the right thigh is 4 cm
greater in circumference than the left thigh and is tender. His temperature is 37.8.
Which of the following is the reason for his presentation?
A. Arterial embolus
B. Cellulitis
C. Deep vein thrombosis
D. Edema
E. Drug interaction

10. 55-year-old man, history of appendicectomy and cholecystectomy presented with 3 wk


history of abdominal pain, distension with tympanic bowel. No vomiting only mild
distension. X-ray shows dilated ascending colon, transverse colon and descending colon, no
air. Rectum is empty and normal looking rectum. No picture X-ray
a. Sigmoid volvulus
b. Adhesive IO
c. CA Sigmoid colon
d. Cecal Volvulus
e. Intussusception

11.

AfraTafreeh.com
Get more from: MplusX Qbank (https://member.mplusx.com/)
Facebook Page: (https://www.facebook.com/mplusxqbank/)

65 year old man, History of appendicetomy & cholecystectomy presented with 3 wk


history of abd pain, x-ray show dilated ascending, transverse and descending colon.
Rectum is empty. (without CT)
a. Sigmoid volvulus
b. Adhesive IO
c. CA Sigmoid
d. Pseudo obstruction
e. Intussusception

12. A 15year old girl having difficulty in concentrating. She constantly fights with father,
recently stopped ballet class – ask why “ what’s the point of all this ? “ , what is the most
important in management?
A)Liase with father
B)Discuss the safety plan with her and her parents
C)Give SSRI
D)CBT
E)Tell school to support her more

13. Lady full term with adequate contraction was monitored on CTG. Everything was
progessing normally. Suddenly there was a deceleration of FHR dropping to 70bpm for 4
mins. What is the next most appropriate step?
a. Stop syntocinon
b. Continue synto at an increased dose
c. Immediate cessarean
d. Ventouse delivery

14. 39 weeks pregnant lady came with labour pain. Synto was given appropriately. Ctg was
done which shows heart rate of 140 which dropped to 70 and came back to 140 in 2min.
asking next appropriate treatment. (Que does not include any basic measure given for her
like Left
lateral position, Oxygen, Fluid)
A. Stop syntocinon
B. Fetal scalp sampling
C. C section
D. Titrate to increase dose of syntocinon
E. Give O2 to mother via face mask

15. A mother came with her two children. Mother's brother has haemochromatosis.
What screening test advice would you offer?

A. Screen only mother


B. Screen mother and kids
C. Ask her to come with her husband
D. Screen only the children

AfraTafreeh.com
Get more from: MplusX Qbank (https://member.mplusx.com/)
Facebook Page: (https://www.facebook.com/mplusxqbank/)

19. Woman with a history of hysterectomy and DVT. Now complaining of hot flushes and
insomnia. What will you give her?
A. Oral low dose estradiol
B. Transdermal estradiol
C. Progesterone only pills
D. HRT
E. Clonidine

20. 48 year old woman complains of heavy menstrual bleeding for last 4 months. She
underwent a hysteroscopy and D&C but the symptoms did not subside. She looks pale
and Her haemoglobin is 8.5 g/dl(exact value). What is the most appropriate
management in this case?
A-oral tranexamic acid during the period
B-northisterone from 15-25 days of cycle
C-mirena
D-implanon

E-COCP

21. 12 yr old girl who is taken from her drug abuse parents and now living in fosters since
she’s 2 year old , she likes toys and very proud of her big collection of toys, plays with
her immaginary friend , eat same foods always and very picky, her carer doesn’t like that
, what is the most immediate danger she is in ?
• 1- drug abuse
• 2-schizophreniform disorder
• 3-sexual abuse
• 4-anorexia nervosa
• 5-OCD

23. T4 (reduce), TSH (normal)


A. MRI brain

B. Thyroid Ab

C. Thyroid scan

AfraTafreeh.com
Get more from: MplusX Qbank (https://member.mplusx.com/)
Facebook Page: (https://www.facebook.com/mplusxqbank/)

35. Woman on HRT on 6 years, well controlled of her symptoms. DEXA scan show femur
-1.7. Vertebra -1.2. How will u manage her?
A. Continue same treatment

B. Cease HRT

C. Add vitamin D and calcium

D. HRT Change to alendronate

E. Add alendronate

36. old patient who drinks about 10-12 standard drinks per day ,underwent some
surgery. on 3rd post op day he developed agitation ,O2 saturation 88%. what next ?
1-blood alcohol
2-blood glucose
3-CTPA
4-Chest X-ray
5-Urine RE

37. Study on Aspirin effects on preventing MI on 100 people. If those who took aspirin of the
100 ppl only 1 person had a MI. Of those who didn't take aspirin 2 ppl had MI. What is the
increase in relative risk in people not taking aspirin?
a. 1%
b. 10%
c. 100%
d. 200%

38. Old women with atrial fibrillation, and abdominal pain, vomiting. looked like
mesenteric ischemia. what investigation to do in addition to CT abdomen
a. Serum lipase
b. Serum lactate
c. Abdominal usg
d. CT scan

40. Patient with COPD was admitted for 3 days in hospital. bronchodilator was given 8
hourly, ipratropium bromide, and oral prednisolone. sputum was of thick yellow color.
Despite the treatment, he still has severe respiratory distress but he is alert. Arterial
blood gas examination are as follow:
pH - 7.35 (7.36-7.44)

AfraTafreeh.com
Get more from: MplusX Qbank (https://member.mplusx.com/)
Facebook Page: (https://www.facebook.com/mplusxqbank/)

PCO2 - 50 (30-45)
PO2 - 80 (80-100)
HCO3 - 35 (22-26)
What is the next appropriate step?
a. CPAP
b. bronchodilator 4 hourly
c. change antibiotic
d. change to IV hydrocortisone
e. arrange refer to ICU for intubation

42. Gout scenario, now presents with acute symptoms. Lab results show urate level little
increase and creatinine is 1.8 (Normal 0.6 – 1.2) not mention renal failure just give
creatinine value. What to give?
A. Prednisolone
B. Codeine
C. Naproxen
D. Allopurinol
E. Colchicine

43. Pt with RA she takes ibuprofen and methotrexate to control her disease for 5 years.
patient complains of some symptoms, and her labs are given
protein- 9 (6-8)
albumin - 4 (3.5-5.5)
ALT - >150 (7-56)
AST - >150 ( 10-40)
GGT - >150 (0-30)
What is the cause of her symptoms?
A- Methotrexate induced hepatitis
B-ibuprofen induced hepatitis
C-Autoimmune hepatitis
D-Viral hepatitis

48. Alcoholic patient present with ascites, gynecomastia, spider nevi, parotid gland
enlargement and flapping tremors and confusion (not mention about abdominal pain). Vitals
given. What investigation to do?
a. Ammonia level
b. Full blood examination
c. Liver function test

AfraTafreeh.com
Get more from: MplusX Qbank (https://member.mplusx.com/)
Facebook Page: (https://www.facebook.com/mplusxqbank/)

d. Abdominal scan
e. Abdominal paracentesis

49. Child presented with UTI, on USG the right kidney is smaller than the left. What is the
best investigation to assess renal function of each kidney? Short scenario

a) DMSA
b) DTPA
c) Urine culture
d) CT scan

50. Child with previous URTI, RBS +, Protein + in the urine dipstick test. After two weeks
urine shows only RBC + and from non-glomerular origin. What is the appropriate
investigation?
A. Urine cytology
B. Bladder scan
C. Renal ultrasound
D. Urine culture
E. Serum electrolytes and creatinine
53. Following MVA, patient brought to ED. Vitals are normal. There is slight
hemopneumothorax and trachea is in the midline (CT given no rib fracture). SPO2 is 98%.
There is tenderness on palpation of the ribs. Patient can’t breathe because of the pain. After
giving oxygen with face mask what will u do next?

A. Morphine

B. Endotracheal intubation

C. Intercostal drainage tube


D. Aspiration

58. 56 year old man with mass which extends from umbilicus to xiphisternum. He
looked morbidly obese and had a waist circumference of 110 cm Asking for
appropriate management for the patient.
A. Weight reduction
B. Abdominal binder
C. Hernioplasty
D. Mesh Herniorrhaphy
E. Observation and Reassurance

AfraTafreeh.com
Get more from: MplusX Qbank (https://member.mplusx.com/)
Facebook Page: (https://www.facebook.com/mplusxqbank/)

60. 60 years old man is having a check-up for prostate cancer last two years PSA was
1.5mg/ml, now PSA is 3.8mg/ml, then 12 slides of biopsy done and only one foci
showed adenocarcinoma. Gleason score is 4. How will you manage this patient?
A. TURP
B. Radical prostatectomy
C. Continue surveillance
D. EBRT
E. Androgen therapy

61. An old man with acute onset pain in the lower back pain. weight loss +. Anal
sphincter tone was normal. On DRE enlarged irregular prostate. What next. Slightly
elevated temperature.
A. Chest x-ray
B. CT spine
C. TRUS
D. Bladder scan
E. PSA

65 yr old man with nocturia and long hx of dribbling urine problems for 12 months
with median sulcus palpable smooth prostate ,On examination ,2 fingers above the
pubic symphysis what next
A. Urine cytology
B. PSA
C. Bladder scan
D. CT scan abdomen
E. serum creatinine

68. Mother living in mining town comes with her daughter for normal blood test. Her
daughter’s
test show lead level 0.75 (normal <0.35) what will be your next initial step?

Exact value of lead level

AfraTafreeh.com
Get more from: MplusX Qbank (https://member.mplusx.com/)
Facebook Page: (https://www.facebook.com/mplusxqbank/)

a) Educate mother about environmental risks


b) Assess IQ testing of child

c) Refer for chelation therapy


d) Move the family to another town
e) Urine test to confirm dx

69. Young primigravida delivered a healthy 3250gm baby after an uneventful labour
lasting only four hours. During labour, no analgesics or epidural was required. There
was slight meconium staining. The fetus developed severe central cyanosis, limp and
Respiratory rate of 40 breaths per minute. What is the most important next step to
resuscitate the baby?
A. Bag and mask ventilation
B. Intubation and ventilation
C. Intravenous sodium bicarbonate
D. Intravenous glucose
E. IV infusion of Naloxone

70. Few days old infant presented to you complained with poor feeding and bile
stain vomiting. His Birth was at 38 weeks gestation and weight 2600 g. he is breast
feeding with no immediate post-natal complications. The child has a history of
passing meconium on day 4 of birth. Now He has mild jaundice and abdominal
distension, and is irritable .On examination, abdomen distended and tender. What is
the appropriate diagnosis?
A. Meconium ileus
B. Hirschsprung’s disease
C. Duodenal atresia
D. Necrotizing enterocolitis
E. Volvulus

73.Post-op oliguria. No fever. No catheterization.

A. Abd xray
B. blood culture
C. S electrolyte and creatinine
D. Bladder scan

AfraTafreeh.com
Get more from: MplusX Qbank (https://member.mplusx.com/)
Facebook Page: (https://www.facebook.com/mplusxqbank/)

74.Post-op oliguria. Fever+ . catheterization +


A. Bladder scan
B. Blood culture

C. S. electrolyte and creatinine


D. CT abdomen

75.A girl want go to India low budget trip she has had previos H/o oral polio, 1 MMR
in childhood , dpta 10 years ago. apart from typhoid and influenza what other
vaccines bud u give?
a. MMR
b. mmr polio
c. Polio, mmr, dtpa
d. mmr, dtpa

76.Child with fever and cough, Chest x-ray given. What is the most appropriate next
step investigation?
A. CT
B. Blood culture
C. Pneumococcal pcr
D. Lumbar puncture
E. FBC

77.30yrs old indigenous woman, with history of Mitral Stenosis. Shortness of


breath for 6weeks. Low grade fever, cough. CXR given (couldn’t find the x-ray) on
examination, bilateral basal crept present in both base of lung. What is the
diagnosis?
A. Pulmonary hypertension
B. Pulmonary fibrosis
C. TB
D. Left heart failure
E. Pneumocystis pneumonia

AfraTafreeh.com
Get more from: MplusX Qbank (https://member.mplusx.com/)
Facebook Page: (https://www.facebook.com/mplusxqbank/)

78.Patient came back from Asia and low grade fever and cough for — weeks.
what is the most likely diagnosis? (X-ray given, I think mediastinum is widened)
a. lymphoma
b. TB
c. Viral pneumonitis
d. pulmonary embolism

79.6month old child came for check-up. On questioning about vaccination,


mother said she had only 2 (1 at birth and 2nd on 2months old) child is having
recurrent URTI. On exam, child is alert, T- 37.8 c, clear discharge from nose. What
is the next management in terms of her vaccination?
a. Investigate underlying cause of urti
b. Hbs vaccination now
c. Vaccine after fever subsides
d. Hbs vaccination now and register for catch up vaccination schedule
e. Hbs vaccination now and standard schedule

86.Scandinavian female 35years BMI 30) for first antenatal check-up . What will you
do?M ,KKJ
a. OGTT as soon as possible
b. OGTT @ 24-28 weeks
c. RBS @28weeks

67. A patient of CKD and hypertensive, patient taking ramipril, carvedilol with many
other drugs presents with confusion labs
urea is 18
creatinine 520
raised calcium 3.1
phosphate also raised
sodium 128
potassium 5.5

What is the cause of confusion in this patient?


A. Uremia
B. Hyponatremia
C. Hyperkalemia
D. Hypercalcemia
E. Hyperphosphatemia

AfraTafreeh.com
Get more from: MplusX Qbank (https://member.mplusx.com/)
Facebook Page: (https://www.facebook.com/mplusxqbank/)

82. A 45 yr old lady smoker and htn uses ocps for many years, BP 150/90mmHg now
comes re prescription and well controlled on her menorrhagia and also says doesn't
want to conceive. Don’t want to change cause it has been good for her menorrhagia.
What advice now?
Nearly exact scenario
A. provided that she stop smoking then can give ocps
B. Use implanon
C. Use copper iud
D. Use progestrogen only pill
E. Use condoms +ocps

84.A indigenous boy had sex with new sex partner. After a few days he found
urethral discharge from his penis. What specimen will help you make a diagnosis of
the STI?

Nearly exact scenario


a.1st catch urine chlamydial PCR
b. Mid-stream urine chlamydial PCR
c. urine culture
d. Urethral swab gram stain and culture

86.Scandinavian female 35years BMI 30 (consumption of snus+ → can develop DM


type2) for first antenatal check-up . What will you do?
d. OGTT as soon as possible
e. OGTT @ 24-28 weeks
f. RBS @28weeks

89.Insufficient breast milk production, reason?


a) insufficient fluid intake
b) not putting for breastfeeding frequently and sufficiently
c) not enough time latching during feeding
d) exhaustion
e) poor positioning

AfraTafreeh.com
Get more from: MplusX Qbank (https://member.mplusx.com/)
Facebook Page: (https://www.facebook.com/mplusxqbank/)

92.24 year old male with a history of fall on an outstretched hand with pain and swelling. X-
ray was given, What is the best treatment option for this patient?
a. Crepe bandage
b. Plaster cast
c. Analgesics
d. Compression screw
e. Plate fixation

93.7 day old baby presented with jaundice since 4 days of life, his birth was at term, not
complicated and he is breastfeeding, serum bilirubin was 240 (normal <200) with direct
120(sure) his liver is palpable 1cm below the costal margin. What is the most appropriate
next step?

A. Thyroid function test


B. USG for liver and biliary tree
C. Coomb test
D. Stop breastfeeding and follow-up
E. Phototherapy

94.Young man after a quarrel had a fracture of floor of eye what is the most consistent
symptom with that?
a-Conjunctival haemorrhage
b-loss of visual activity
c-anaesthesia around the cheek

AfraTafreeh.com
Get more from: MplusX Qbank (https://member.mplusx.com/)
Facebook Page: (https://www.facebook.com/mplusxqbank/)

d-Cant open the mouth completely


e- Epistaxis

101.He sustained wound on that arm 1 month back but it was healed. Drinks 1
glass of wine every day and smokes 10 cigarettes a day. Which of the following is
the most common cause?
A. Farm worker
B. Chronic infection
C. Genetic
D. Smoker
E. Alcoholism

104.92-old patient found unconscious at home brought by son to emergency department.


the patient was taking tablets methadone, oxycodone , others , patient has constricted pupil
on examination which of the followings is the likely cause?
a.methadone
b.oxycodone

105. Girl 14 years history of weight loss BMI 15. Her school performance is good and
recently fall in grade. Which of the following will have in the history of her?
A. Episodic fatigue (exact words)
B. Sleep disturbances
C. not going around with friends
D. Feeling inferiority
E. idea of guilt

106. A man is brought by policemen after beaten his friend during an argument, he is
intoxicated for alcohol what in history will be important to say personality disorder
A.History of fight with colleague two weeks ago
B.History of cruelty to animals in childhood
C.Alcohol and drug abuse

AfraTafreeh.com
Get more from: MplusX Qbank (https://member.mplusx.com/)
Facebook Page: (https://www.facebook.com/mplusxqbank/)

111.A woman came for interview asked about appetite she said one doesn't get cheese from
moon and I came by bus What will be has in her history?
A.Disorganized behavior
B.Depression
C.Delusion
D.Depersonalization
E.Dissociation

114.Restless leg syndrome scenario. What is the next investigation?


a.Sleep studies
b.Nerve conduction test
c.Iron studies

115.Male old patient usually come with her wife for medical assessment. This time case of
his prostate cancer result. He is only invited to the consulted room. For breaking the bad
news which of the following is most appropriate to start?
A. “do you want your wife to be with you now?”
B. “I will discuss with your wife first”
C. “it’s better you are alone”
D. “how much you want to know regarding this result”
E. Don’t remember this one
116.14 years old girl lives with family has now become sexually active comes to you for
contraception advice and tells you not to inform her parents. What will u do?
A. Give her prescription of contraception
B. Ask her about identity and age of her sex partner
C. Tell her she needs parents’ permission
D. Inform parents
E. Tell her to use condom to her partner

119.man comes with induced vomiting after binge eating .What will have in his family
history?
A.obesity
B.OCD
C.depression

AfraTafreeh.com
Get more from: MplusX Qbank (https://member.mplusx.com/)
Facebook Page: (https://www.facebook.com/mplusxqbank/)

122. A postmenouposal lady with the fracture vertbrae and DEXA -2.5 femour neck. Her Vit D
level is normal. She has severe GERD and is on pantoprazole for it.
What is best ttt?
- alondorate
- zolandroic acid
- HRT
- raloxifen
- setronium

123.all options are same Another Q does not include zolandronic acid (past history of refex
disease and oesophageal scarring)

125. a patient presents with 3 months history of nausea and epigastric


discomfort he has icteric sclera he consumes 50-60 grams of Alcohol. Lab
investigations were given all liver enzymes including total and conjugated
bilirubin were raised and serum lipase was also raised on Usg there was
dilatation of intrahepatic channels not extrahepatic no other abnormality what
you will do?
ERCP
CT abdomen
MRCP
Endoscopic ultrasound

126. 55 yrs old man with no family history of prostate cancer comes for
screening advice and counselling . He insist he wants a DRE.
A digital rectal exam showed a normal sized prostate with normal non nodular
sulci. What is the next important step
A. come after symptoms appear
B. PSA
C .TRUS
A. Abdominal usg

127.13 years old boy violated to school rules,destroy furnitures , rude to


teachers and sisters .No problem in primary school and childhood.What is the
diagnosis?
A.conduct
B.ADHD
C.ODD

AfraTafreeh.com
Get more from: MplusX Qbank (https://member.mplusx.com/)
Facebook Page: (https://www.facebook.com/mplusxqbank/)

128.An obese man had hypertension and family history of DM.On visits,BP-
130/85 . FBS-7.8. What will u do?
A.repeat FBS
B.repeat RBS
C.commence metformin
D.commence insulin

130. A middle-aged man with a history of a chronic lung disease, "they didn't specify which",
presents with an increasing dyspnea over the past 12 hours. He looks well, he can speak in
phrases, the dyspnea has been tolerated well for the past few years. Now he has an SpO2 of
85% on 8L oxygen. What's the initial investigation you'll request?
A. ABG.
B. Blood culture.
C. Chest xray.
D. CT chest.

Best option spirometry

131. 2 yr child girl presented in ED with burn hot water on her face ,resident took picture to
discuss treatment with colleague .Appropirate step of Resident?
Can use in ED

Can share with colleague Share only after mother consent


Can use as medical purposes
Can use in multimedia

132. Old age pt with vomiting and abdominal distention and passing flatus last 12 hours
unremarkable history of surgery or other diseases asking about the Dx?
-small Bowel obstruction
-large bowel obstruction
-Pseudo obstruction
-volvulus

Similar pic

133. your in a rural area and 4 cases of trachoma come to you then you find out extra 20 how
can you treat acute indexes ?

AfraTafreeh.com
Get more from: MplusX Qbank (https://member.mplusx.com/)
Facebook Page: (https://www.facebook.com/mplusxqbank/)

a-hand washing
b-azithromycin
c-doxycyclin
d-don’t do anything
e-penicillin

134. Short term dementia MMSE given 24, wants to change will. Daughter says it would be
illegal to do so
bA. She can change her will
B. She can't
C. Refer for neuropsychiatric review
D. Ask lawyer

136. . 52 years old woman with incontinence of urine during coughing and laughing. Her
urodynamic study shows that her incontinence occurs during low volume study. On
examination she had mild rectocele. Her symptoms showed some improvement after pelvic
floor exercise but she could not satisfied this result. What will your appropriate step
management?
A. Bladder neck suspension
B. Topical Estrogen cream
C. Weighted vaginal cones
D. Imipramine

137.
75 yr old man living alone found to have increasing forgetfulness. Diagnosis? Exact picture
and options
A. cerebral infarct
B. cerebral tumor
C. cerebral abscess
D. cerebral hemorrhageis
E. subdural haematoma

139. Young male came for infertility ,scanty hairs on body, small testis, height 183 cm, what you
going to do to reach diagnosis
Serum testosterone
Chromosome analysis

140. 35 yr old lady presented to you for breast CA screening as one of her paternal aunt is
with Breast and ovarian ca. What will u advise her?
A.2 yarely mammography

AfraTafreeh.com
Get more from: MplusX Qbank (https://member.mplusx.com/)
Facebook Page: (https://www.facebook.com/mplusxqbank/)

B.2 yarely ultrasound


C. Genetic risk screening
D.BRCA gene screening
E. 6 monthly self-breast examination

141. A patient has Alzheimer dementia and Ca colon. the surgeons have decided her best
treatment is bowel resection with colostomy. One of her daughters asks you about the
implications of this procedure considering her mother cannot take care of herself (pls note, not
mentioned that daughter is refusing surgery). Patient MMSE is 20/30. You talk to patient and she
has given a consent for operation. What is your next most appropriate course of action ?
a) you will proceed for operation as she has given consent
b)you will not operate as pt is not capable of giving consent
c)arrange a meeting between daughter/family and the treating doctors for her best management
d) provided her daughter has power of attorney, she can give consent for not operating her
mother

142.27 years old come with green brown nipple discharge . On examination ,no palpable mass
and discharge is expressed from 3 opening of nipple.What is the most appropriate test to get
diagnosis?

A.biopsy of nipple
B.ductogram
C.FNAC of nipple
D.USG
E.mammogram

143.woman is brought by neighbor because her husband push her from stair.on
examination ,there are many bruises and only soft tissue injury.She is afraid to go backto
husband .What will u do?
A,encourage to go to police
B.refer to local domestic violence service
C.call her relative to pick her up
D.advixe to live with neighbor

144. Pharmaceutical company approaches u with results of comparison between 2 drugs for
hyper cholesterolrmia, wat is d important question u wil ask

A- wether same dose of dugs wer used


B- wether 2 grp s contained d same no of people
C- how did u equalise d dietary factors

1. What would be the best indicator for development of further violent


behavior in a demographic?

a. Past h/o violent behavior

b. H/o previous imprisonment

AfraTafreeh.com
Get more from: MplusX Qbank (https://member.mplusx.com/)
Facebook Page: (https://www.facebook.com/mplusxqbank/)

c. juvenile delinquency

2. A photo of a newborn with rash and cardiac catheterization was given (not
sure). What will you test in mother?

a. Renal function test

b. Liver function test

c. Thyroid function test

3. After starting ocp, intermittent spotting occurs, cause asked


a. endometrium not forming completely
b. ovarian follicle response
c. others

4. An old lady who staying alone come to ER with confusion and irritability after
staying for 4 hrs she become stable but still irritable and not cooperative. When
you examine her, you found the bruises on her upper limbs. What would be the
cause of her bruises?
a. elderly abuse
b. due to alcoholic induced vitK deficiency
c. subclinical haemorrhage?? (not sure)

A man had been on anti-Parkinson drug for 9 years. Recently, his wife
complains of psychosis and was prescribed 10mg Haloperidol. After two weeks,
wife complains of abnormal movement and inability to feed himself. What’s the
cause?
A) Worsening Parkinson
B) Lewy body dementia
C) Haloperidol

A young man has a syncopal attack while weight lifting. He has had similar
episodes twice before. His father died of cardiac disease. What is the next
most appropriate management in diagnosis?
a) Holter monitor
b) EKG
c) Stress test
d) BP in supine & lying down
e) CT scan

AfraTafreeh.com
Get more from: MplusX Qbank (https://member.mplusx.com/)
Facebook Page: (https://www.facebook.com/mplusxqbank/)

An old man with onset pain in the lower back was given indomethacin. With
little weight loss, anal sphincter tone was normal. On DRE, prostate was bit
enlarged and smooth. Patient now complains of hematuria. What is dx?
A. Indomethacin induced hematuria
B. testicular cancer
C. abdominal cancer
D. acute pylonephritis
E. kidney failure
A girl is taken to the hospital after being found unconscious. On her h/x, she had
drank petrol from a soft drink container, 500mg paracetamols/aspirin & 10
standard drinks. What is your immediate tx?
A. give naloxone
B. give N-acetyle infusion
C. Take to surgery
D. ABG
E. gastric lavage
A man was taken to you from prison after being aggressive and threatening to
kill himself. He is convicted and currently serving time for trying to kill his
parents because he thinks they have been replaced by double. Upon reviewing
the h/x, you realize the man has been diagnosed with schizophrenia as a child
and given dexamethasone. The warden calls and asks for your next step in
management. What do you do?
F. Send the man back to prison
G. Administer anti-psychotics and send the man away
H. Release the man from custody
I. Send to prison hospital
J. Send to local psych unit
65 year old man undergoes surgery for ruptured abdominal aneurysm and has
no other problems. Upon examination, he shows no signs and reflexes are
normal. What is your next step in management?
F. Colloid solution
G. 3% saline
H. 0.9% solution
I. Transfuse pack of red blood cells
J. Oral iron

AfraTafreeh.com
Get more from: MplusX Qbank (https://member.mplusx.com/)
Facebook Page: (https://www.facebook.com/mplusxqbank/)

Old lady comes to you stating that she is feeling unwell. According to her h/x,
she has HTN, diabetes, hypothyrodism and cardiac disease for which she takes
io-thyroxine, metformin, verapamil, elondril? and thyroxine. Which tx will
alleviate her discomfort?
F. Cease metformin
G. Cease verapamil
H. Cease elondril
I. Cease thyroxine
J. Increase thyroxine
Huge eye pic: Patient presented with acute glaucoma what is the first thing you
should give :
F. acetazolamide
G. timolol
H. pilocarpine
I. steroids
J. ice packs

2. A man presenting with abd pain and distension for 3days, constipation for 2
days. Vital sign stable apart from and distended with sluggish bowel sound. Abd
Xray given(shows Small bowel obstruction) After 2days, gave IV antibiotics and
nasogastric aspiration. Not relieved symptom. What is next management?

A. continue the same treatment

B. IV N/S and hydrate

C. colonoscopy

D. Surgical

E. Nasal flatus tube

1. A man accident at car accident by head on collision, GCS-10, vital sign stable.
given CT head shows EDH picture.
A.SAH

B.SDH

AfraTafreeh.com
Get more from: MplusX Qbank (https://member.mplusx.com/)
Facebook Page: (https://www.facebook.com/mplusxqbank/)

C.EDH

5. 30yrs old F, came with malaise, fatigue, palpitation, wt loss 4kg,HR-


120/min,regular.Thyroid is palpable.TSH-0.1 low.Thyroid scan increase uptake.
She was given propanolol. What is next step of management?
A.carbimazole

B.radio iodine

C. Subtotal thyroidectomy

D.prednisolone

6. Coxiella burnetii- what causes infection-


a. factory worker
b. saw factory worker
c. dairy farmer

7. 12 week old pregnant lady was brought to the emergency department with
vomiting, abdominal pain and severe bleeding for the past 4 hours. She also had
2 fainting attacks. Her blood pressure is 80/40.She was normal till before this
episode and USG at 11 weeks showed normal nuchal translucency. Which is the
most likely diagnosis?
Incomplete abortion
Complete abortion
Abruption placenta
Ectopic pregnancy rupture
Hydatidiform mole
1.
2. Pt BP 110/50 mmHg, on exam a pulsatile abdominal mass felt in midline, with
severe pain radiating to back. Most appropriate next?
a) Urgent laprotomy
b) USG
c) CT
d) Aortogram
e) NS

3. Milestones development of a 2 year old girl.


Can describe 4 colours
Can use 4 syllable sentence

AfraTafreeh.com
Get more from: MplusX Qbank (https://member.mplusx.com/)
Facebook Page: (https://www.facebook.com/mplusxqbank/)

Knows family name


Can use 2 personal pronouns

4. Child got superficial abrasions and lacerations after falling in garden bed..he
has h.o 2 DTPa vaccines..most appropriate step after cleaning the wound?
A)give tentnus toxoid n topical antibiotic cream
B)give tetnus toxoid and oral penicillin
C)give tetanus toxoid n immunoglobulins
D)DTPa and booster in 2 months
E)DTPa and immunoglobulins

5. An 18 year old girl presents with epistaxis, malaise and tiredness for months.
No other symptoms. Hb is 8.5, serum ferritin is 35, INR is 1.5. Calcium is 1.9.
Long question. What to do next ?
A. Blood transfusion
B. IV calcium
C. Injection vitamin k
D. Fresh frozen plasma
E. Vitamin D tablets

6. CT scan chest MVA with chest pain and difficulty breathing. Decreased breath
sounds bilaterally and dullness(hemo) to percussion. Asked about the cause
of pain.

a) Fractured ribs
b) Hemothorax
c) Pneumothorax
d) Pneumomediastinum

7. Pacemaker syndrome with difficulty in breathing with faint heart sounds


investigation of choice needed.
A. Chest xray.
B. CTPA.
C. ECG.
D. ECHOCARDIOGRAM.
E. SERUM TROPONIN

8. Lady comes on 10th postpartum day with pain due to perineal tear.Now on
examination there is perineal wound that is 2 cm long,1cm wide and 1 mm
deep.It was clean with no discharge.
What to do next?
Apply local antibiotics
Put on oral antibiotics
Simply keep the wound clean
Suture the wound under local anaesthesia
Suture the wound after cutting the edges

AfraTafreeh.com
Get more from: MplusX Qbank (https://member.mplusx.com/)
Facebook Page: (https://www.facebook.com/mplusxqbank/)

9. pregnant woman at 12 weeks gestation presents with heavy bleeding had an


ultrasound Nuchal translucency at 11 weeks Abdomen rigidity, tenderness.
Maternal hypotension and tachycardia. Asking diagnosis
a. incomplete abortion
b. threatened Abortion
c. Ectopic pregnancy
d. Placenta previa
e. molar pregnancy
10.
11. Patient’s RPR 1 year ago 0, then 6 months back was 1:256, now its 1:2.
antibodies were positive 6 months back and now.
a. Patient is syphilis carrier
b. Patient has syphilis now
c. Patient had syphilis 6 months back

12. 10 minutes weakness of right side of body with some speech trouble. Return
to normal with no carotid bruit and blurring of vision. What it could be?

a. Lacunar infarct
b. Vertebro basilar
c. ischaemia caotid artery
d. hypoglycemia
e. hyperglycemia

13. Cyclist fell on outstretched hands and painful wrist with movement restriction
X ray done. Appropriate tx? Xray of scaphoid fracture

Crepe bandage
Screw fixation
Slab and early mobilization
Plaster cast
Plate

14. patient presented with macroscopic hematuria. 6 months back he diagnosed


as c-ANCA positive vasculitis causing hematuria and started on Prednisolone
and Cysclophosphamide. The RBCs on examination are 20% dysmorphic and
80% normal, what is the most appropriate next step?
a. Repeat renal biopsy
b. Stop cyclophosphamide
d. CT abdomen
d. Renal USG
e. Cystoscopy

15. Heart failure case man with AF on Ramipril, metoprolol, digoxin. Went on trip
for 2 weeks and stopped his medication. Now with bilateral oedema up to
knee in legs. And ankle swellings Lungs is clear. And the patient is not
dyspneic,

AfraTafreeh.com
Get more from: MplusX Qbank (https://member.mplusx.com/)
Facebook Page: (https://www.facebook.com/mplusxqbank/)

What is the most appropriate management?


A- Frusemide
B- Recommence all his medications
C- Ramipril
D- Digoxin

28.patient was on sertraline . 1 week ago due to some pain she was prescribed
tramadol. Now he has developed tremor,agitation , hyperreflexia, temp was
normal. what will you do?
a. stop sertraline and review after 3 days
b. stoptramadol and review after 1 day
c. give bnz and review after 1 day
d. give betablocker and review after 1 day
e. admission to the hospital
29

30.patient with panic disorder came with palpitation , restlessness . she was
taking some benzodiazepine which she stopped 2 days back . what is the cause
a.generalized anxiety
b.benz withdrawl
c. panic attack

31. 3rd pod knee surgery . pt is a smoker and takes 2 beer everyday. Now
developed agitation , confusion . what will be most important intervention(no
mention of oxygen saturation)
a. iv heparin
b.benzo
c. oxygen by mask

33. 55yr male,smoker,h/o surgery for ca stomach;swelling in the lateral neck fro 3
weeks,non tender,smooth,sof,free ffrom overlying skin.oropharyngealexamination
ormal.whts Dx?

a.branchial cyst-
b.pleomorphic adenoma of parotid-
c.submandibular duct stone
d.metastasis

AfraTafreeh.com
Get more from: MplusX Qbank (https://member.mplusx.com/)
Facebook Page: (https://www.facebook.com/mplusxqbank/)

37. Woman with a history of hysterectomy and DVT. Now complaining of hot flushes
and insomnia. SSRI was not helpful in relieving symptoms..What will you give her?
A. Oral low dose estradiol
B. Transdermal estradiol
C. Progesterone only pills
D. HRT
E. Clonidine

38. Well controlled Diabetic with mild renal impairment asking about risk for baby?
Asymptomatic patient {Creatinine 120(normal value was 80-110 given in question)}
A. IUGR
B. Macrosomia
C. Renal agenesis
D. Intrauterine fetal demise
E. low Birth weight

39. 15year old girl having difficulty in concentrating. She constantly fights with father,
recently stopped ballet class – ask why “ what’s the point of all this ? “ , what is the
most important in management?
A)Liase with father
B)Discuss the safety plan with her and her parents
C)Give SSRI
D)CBT
E)Tell school to support her more

40. 65 yr old man with nocturia and long hx of dribbling urine problems with
median sulcus palpable, smooth prostate what most appropriate next
A. Urine cytology
B. PSA
C. Bladder scan
D. CT scan abdomen
E. IVU
41. young mother with 10wks old attending clinic many times within 2 weeks
complaining baby cries a lot and difficult to settle. Baby is growing well and
all normal. What will u find in her history for diagnosis?

Premorbid personality
Whether she consider harming the baby
Past obstetric history
Thought about baby’s behaviour
Confidence of caring baby

43. child 6 years old with history of asthma presented with upper respiratory tract
infection, urine examination showed very high serum glucose and ketone body of
2+. What is the most appropriate test to follow up this child?
A. HbA1C
B. Serum creatinine and electrolyte
C. FBS
D. OGTT
E. ABGA

AfraTafreeh.com
Get more from: MplusX Qbank (https://member.mplusx.com/)
Facebook Page: (https://www.facebook.com/mplusxqbank/)

44. A 46 years female presented with lower abdominal pain for 2 days. Her last
menstrual cycle was normal n three weeks ago. On examination there is mild
tenderness n fullness in right adenexa. Usg shows cystic mass of 10 cm in right
ovary whats the diagnosis??
A. Krukunberg tumor
B. Luteal cyst
C. Serous cystadenoma D. Cystoadenosarcoma E. Mucinous cysadenocarcinoma

46. Business man got bankrupt, many financial problems, complaining of late
insomnia, lack of pleasure, loss 15 kg from his weight through 3 months, many other
symptoms of depression, his family concern about him and the patient refused to
take any medication as he doesn’t believe he is sick, he admit he is tired and
exhausted but not depressed, what’s this called?
A Denial (ans)
B Reaction formation
C Displacement
D Rationalisation

47. Scenario of cancer head of pancreas .. upper abdominal pain , jaundice , weight
loss,lab value given with raisef ALP, raised GTT, normal ALT, and normal
AST…investigation asked next?
a. CT abdomen
b. ERCP
c. MRCP
d. USG abdomen

48. 10) 36 years old patient who drinks about 10-12 standard drinks per
day ,underwent knee replacement surgery.on 3rd post op day he developed agitation
and dyspnoea,O2 saturation 88%.RR 30..what next ?
1-blood alcohol
2-blood glucose
3-CTPAans
4-XRAY CHEST

49. 58yrs old man, drag left foot, reflexes on left lower limb are increased, planter
flexion & dorsiflexion are 4/5, weak ankle movements, equivocal plantar reflex, upper
limb and face are normal exam.Lesion site
a. Common perineal nerve
b. Cervical spinal cord
c. L5,S1 nerve root
d. Cerebral cortex
e.Brain Stem

50. Patient with deep jaundice, massive ascites, and confused complaining of
abdominal pain due to tense ascites. Flapping tremors and hepatic encephalopathy
feature. Liver is difficult to palpate. Compaint of high fever 40’C. What is your next
appropriate step?
A- diagnostic paracentesis
B- Blood culture

AfraTafreeh.com
Get more from: MplusX Qbank (https://member.mplusx.com/)
Facebook Page: (https://www.facebook.com/mplusxqbank/)

C- USG
D- Fluid restriction
E- IV antibiotics

51. Mother living in mining town comes with her daughter for normal blood test. Her
daughter’s test show lead level 0.72 micromoles/L ; what will be your next
appropriate step?
A. Assess IQ testing of child
B.Educate mother about environmental risk
C. Refer for chelation therapy if >10 and severe symptomatic. (chelation
if >/=45microgram/l or >/= 2.2micromol/L)
D. Move the family to another town
E. Urine test to confirm dx

53. man travelled asia. 2 weeks history of cough n mild fever and epigastric pain but
no rigidity, x ray was there,because of his presentation…
a.tb
b.viral pneumonia
c.pneumonia pneumococcal
d.fungal infection

54. 33) A woman c/o forgetfulness (age 58). He forgets where he keeps his
things,easily gives up task .during cognitive assessment,she becomes upset and
didn’t want to do it.What to check in mental state examination (MSE)?
a) Orientation
b) Hopelessness
c) Hallucination
d)insight

55. Old man came with his wife.he gets confuse sometimes and forgetful. He still
drives, sometimes at night .he stopped reading newspaper.mmse 26/30 .ct scan of
brain normal.most appropriate management?
a- Cease driving at night
b- Repeat mmse after 3 months
c- MRi of brain
d- encourage physical activities

56. woman comes to your clinic. She was prescribed trifluoperazine for her condition.
She was taking it for 3 years with improvement of her condition. She says that she

AfraTafreeh.com
Get more from: MplusX Qbank (https://member.mplusx.com/)
Facebook Page: (https://www.facebook.com/mplusxqbank/)

discontinued taking her medication for the last 3 weeks because her doctor was not
present for he was in trip, she also said that trifluoperazine makes her hand or some
muscle part stiffy, rigid, and restless.Now she presents with voices in her head. What
is the most appropriate initial choice in management?( 516 jm)
A-trifluoperazine
B-resperidone
C-quitapine
D-respa depot
E-stop trifluoperazine

57. ) 30yrs old indigenous woman, with history of Mitral Stenosis. Shortness of
breath for 6weeks. Low grade fever, cough. CXR given (see below; looks like this
one) on examination, bilateral basal crept present in both base of lung. What is the
diagnosis?
A. Pulmonary hypertension
B. Pulmonary fibrosis
C. TB
D. Left heart failure (Short duration of dyspnea)
E. Lymphoma

58. old man was bought to ED for taking multiple drugs among which was
.p/c ,methadone,oxycodone,ibuprofen naloxone was given to the patient.on
examination pin point pupil.what level is important to measure in his management
a.methadone
c.ibuprofen
d.paracetamol

59. A 29-year-old woman has been treated with sertraline 50mg dailyfor the past five
weeks. She had initially presented with a four weekhistory of bouts of crying,
insomnia, tiredness and weight loss as wellas a general loss of interest in her family,
friends and her job. Hersymptoms had followed the break up of a longterm
relationship andhave improved with the treatment. She is now complaining
ofpersistent abdominal discomfort over the past month. Which one ofthe following is
the best initial management strategy at this stage?
A. Switch to another selective serotonin reuptake inhibitor (SSRI)
antidepressant.

AfraTafreeh.com
Get more from: MplusX Qbank (https://member.mplusx.com/)
Facebook Page: (https://www.facebook.com/mplusxqbank/)

B. Switch to a selective noradrenaline reuptake inhibitor (SNRI)


antidepressant.
C. Switch to a tricyclic antidepressant.
D. Stop the sertraline immediately
E. Augment the sertraline with a mood stabiliser.

60. 55-year-old man, history of appendicectomy and cholecystectomy presented with


3 wk history of abdominal pain, distension with tympanic bowel. No vomiting only
mild distension. X-ray shows dilated ascending colon, transverse colon and
descending colon, distended until sigmoid colon where there is cutoff of colonic gas .
Rectum is empty and normal looking rectum. No picture X-ray. Asking diagnosis
a. Sigmoid volvulus
b. Adhesive IO
c. CA Sigmoid colon ans
d. Cecal Volvulus

e. Intussusception
61. Lady full term with adequate contraction was monitored on CTG. Everything was
progessing normally. Suddenly there was a deceleration of FHR dropping to
70bpm**** for 4 mins. What is the next most appropriate step?
a. Stop syntocinon ans
b. Continue synto at an increased dose
c. Immediate cessarean
d. Ventouse delivery
62. 39 weeks pregnant lady came with labour pain. Synto was given appropriately.
Ctg was done which shows heart rate of 140 which dropped to 70 and came back to
140 in 2min. asking next appropriate treatment. (Que does not include any basic
measure given for her like Left
lateral position, Oxygen, Fluid)
A. Stop syntocinon ans
B. Fetal scalp sampling
C. C section
D. Titrate to increase dose of syntocinon
E. Give O2 to mother via face mask
63. Young woman comes to you complaining of heavy menstrual bleeding with pain.
She describes that clots of blood are lost in the first few days of menses with severe

AfraTafreeh.com
Get more from: MplusX Qbank (https://member.mplusx.com/)
Facebook Page: (https://www.facebook.com/mplusxqbank/)

back pain. You ask her for some investigations that need to be performed. She gets
irriated and says she just wants a treatment. Which of the following is the apporpraite
next treatment ?
a. Tranexemic acid from day 1 of menses
b. Mefanimic acid from day 1 of menses ans
c. Mirena
d. Oral contraceptive pills
64. A study is conducted to check the efficacy of antibiotics prior to suturing wound
what is increase in absolute risk reduction?
Values were 2 4
98 96
Options were
a- 1%
b- 2 % ans
c- 33%
d- 50%
e- 98%
65. Woman whose son is a drug addict, beats her every day. She is afraid and
presents to you. What will u do for the safety of the woman?
A. Inform her that you are obliged to inform the police
B. Go to the police
C. urge her to go to refugee shelter ans
D. Advise her to go support group
E. Send the son to drug addiction service

66. Scenario with female patient always angry, and can’t control herself, unstable
relationships. Self-harm was there, treatment was asked
a. anger management
b. dialectal behavioral therapy
c. Psychoanalytical therapy
d. Interpersonal therapy

AfraTafreeh.com
Get more from: MplusX Qbank (https://member.mplusx.com/)
Facebook Page: (https://www.facebook.com/mplusxqbank/)

67. Woman after death of her husband due to prostate cancer, lives alone ,6 weeks
later presented with loss of appetite, thinks he died because of her infidelity. She
sleeps badly(exact) and had similar episodes after the death of her child. Prompt
treatment asked.
Risperidone
Venlafaxine
ECT
Mirtazapine

70. Old women with atrial fibrillation, and abdominal pain, vomiting. looked like
mesenteric ischemia. what investigation to do in addition to CT abdomen
a- Serum lipase
b- Serum lactate ans
c- Abdominal usg
d- CT scan

71. Pt with RA she takes ibuprofen and methotrexate to control her disease for 5
years. patient complains of some symptoms, and her labs are given
protein - 9 (6-8)
albumin - 4 (3.5-5.5)
ALT - >150 (7-56)
AST - >150 ( 10-40)
GGT - >150 (0-30)
What is the cause of her symptoms?
A- Methotrexate induced hepatitis ans
B-ibuprofen induced hepatitis
C-Autoimmune hepatitis
D-Viral hepatitis

74. Child with previous URTI, RBS +, Protein + in the urine dipstick test. After two
weeks urine shows only RBC + and from non-glomerular origin. What is the
appropriate investigation?
A. Urine cytology

AfraTafreeh.com
Get more from: MplusX Qbank (https://member.mplusx.com/)
Facebook Page: (https://www.facebook.com/mplusxqbank/)

B. Bladder scan
C. Renal ultrasound
D. Urine culture
E. Serum electrolytes and creatinine

75. Patient with parkinsonism like features (cogwheel rigidity and difficult
swallowing), history of taking risperidone and now visual hallucination. His cognitive
function is impaired. What is the most likely diagnosis?
a. Dementia
b. Lewy body dementia ans
c. Drug interaction
d. Cerebral infarct
e. Parkinsonism

76. Middle age man present with retrosternal chest pain. This is preceded by
vomiting. On examination, reduced breath sounds in lower lobes of lungs and
dullness over these. Next appropriate investigation?
a. CXR
b. Gastrograffin swallow
c. CTPA
d. Non contrast CT chest
e. ECG
• 77. 65-year-old man presented with loin pain, urine examination showed
hematuria. What is the most appropriate next step? Exact age
A-X-ray
B-abdominal ultrasound
C-CT abdomen
D-MRI
E-urine culture

78. 56 year old man with elongated mass which extends from umbilicus to
xiphisternum. Asking for appropriate management for the patient.
A. Weight reduction

AfraTafreeh.com
Get more from: MplusX Qbank (https://member.mplusx.com/)
Facebook Page: (https://www.facebook.com/mplusxqbank/)

B. physiotherapy
C. Hernioplasty
D. Mesh Herniorrhaphy
E. Observation and Reassurance

81. An old man with acute onset pain in the lower back pain. weight loss +. Anal
sphincter tone was normal. On DRE enlarged irregular prostate. What most
appropriate next?

A. Chest x-ray
B. CT spine
C. TRUS
D. Bladder scan
E. PSA

89. A indigenous boy had sex with new sex partner. After a few days he found
urethral discharge from his penis. What specimen will help you make a diagnosis
of the STI?

Nearly exact scenario


a.1st catch urine chlamydial PCR
b. Mid-stream urine chlamydial PCR
c. urine culture
d. Urethral swab gram stain and culture

http://www.sti.guidelines.org.au/syndromes/urethritis-male#clinical-
presentation

90. Woman 51 years of age presents after menopause for the last 8 months. She
complains of irritability, moodiness, low libido and hot flushes. Which of the
following would be most appropriate to give to this woman?
a. Combined continuous estrogen and progesterone throughout the cycle
b. Continuous estrogen throughout the cycle

AfraTafreeh.com
Get more from: MplusX Qbank (https://member.mplusx.com/)
Facebook Page: (https://www.facebook.com/mplusxqbank/)

c. Continuous estrogen with progesterone 12 days of a month ans


d. Continuous progesterone with testosterone

91. 15 wks pregnant comes for risk of Down syndrome. What is the diagnostic
test for her(Exact)?
a) amniocentesis
b) CVS
c) nuchal fold thickness
d) serum screening
92. Young couple with infertility. Female with one child in previous marriage and
female investigations were normal, what in history you will ask the male patient
that will be most useful to guide u to cause of infertility?
A. Do u drink alcohol so much? ans

B. Did u shave often?


C. Do you use Marijuana?
D. Do u have chicken pox?
E. Do u often use ectasy and party drugs?
93. Young man after a quarrel had a fracture of floor of eye what is the most
consistent symptom with that?
a-Conjunctival haemorrhage
b-loss of visual activity
c-anaesthesia around the cheek
d-Cant open the mouth completely
e- Epistaxis

94. Indigenous child with chronic suppurative otitis media presents with chronic ear
discharge. Ear swab done showed existence of pseudomonas aeruginosa
organisms. After ear toileting what will u do?
A. Oral Amoxicillin
B. Oral Augmentin
C. Ciprofloxacin ear drop
D. Paramycetin with steroid ear drops
E. Hearing aids

AfraTafreeh.com
Get more from: MplusX Qbank (https://member.mplusx.com/)
Facebook Page: (https://www.facebook.com/mplusxqbank/)

95.

What is the most appropriate management? Exact picture

A. Local excision ans


B. Excision with a 2 cm margin.
C. Fluorouracil cream
D. Review in 12 months

96. 40 years old lady complaining of back pain after gardening On examination,
there was tenderness in the lumbar vertebrae L3 / L4 with limited flexion and
extension. After giving analgesics what is your next line of management?
A. X-ray
B. observation ans
C. refer to orthopedics
D. corset
E. MRI

97. 54 years old lady complaining back pain after lifting heavy object. No
numbness nor sensory loss. Pain down through buttock to thigh. What to give in
addition to analgesia

A- Continue activity
B- Refer to orthopedic
C- Corset
D- Traction
E- Bed rest

AfraTafreeh.com
Get more from: MplusX Qbank (https://member.mplusx.com/)
Facebook Page: (https://www.facebook.com/mplusxqbank/)

98. A 48 years man presents with symptoms of claudication. His BP IS 140/80


and he has 40 pack year smoking history. You assess him and decide he does
not require intervention. He asks you what is the most important step in reducing
his symptoms.
A. Reduce smoking
B. Reduce BP
C. Supervised exercise
D. Atorvastatin
E. Arteriography

99.
He sustained wound on that arm 1 month back but it was healed. Drinks 1
glass of wine every day and smokes 10 cigarettes a day. Which of the
following is the most common cause?
A. Farm worker
B. Chronic infection
C. Genetic
D. Smoker
E. Alcoholism

100. 24 months child verbal milestone?


d. 2 pronouns
101. You are intern. The patient is intubated and one of his family members says
the patient wants to change the will. What should you do?
e. Witness the will
f. Refuse to witness ans
g. Ask surgical register if he can do it
h. Seek legal advice
i. Ask the nearby kins and witness the will

AfraTafreeh.com
Get more from: MplusX Qbank (https://member.mplusx.com/)
Facebook Page: (https://www.facebook.com/mplusxqbank/)

102. year old retired farmer presented with left groin lymph node enlargement.
The biopsy specimen shows squamous cell carcinoma. what is the primary site?
j. anus
k. rectum
l. prostate
m. left leg ans
103. A patient with spiking fever, SOB and was inserted prosthetic heart valves 3
months ago. And 3 blood cultures done dx as due to staph aureus IE, and given
flucloxacillin for treatment. The fever subsides after 2 weeks of treatment, but
SOB still persists, diastolic murmur at the left sternal edge? What will u do for
investigation?
e) CXR
f) troponin
g) blood culture for every week
d)trans thoracic ECHO
104. Pt with smaller kidney on one side, what to do to check individual kidney's
function?
A. DMSA ans
B. DTPA
C. IVU
D. USG
105. Lady will join permanentnight shift.take balanced diet,mildly fair
skinned,participate in outdoor games.come regarding her concern abt vit D as she
will nothave enough day light exposure due to her job.what will be ur course of action
regarding this lady?
a.measure her vit D status
b.add vit D rich food with her balanced diet 9AMEDEX)
.give her vitD supplement
D. Give soralin
E REASSURE her diet will be enough for her

106.2 ecg….1 with normal ecg with potassium level 5.5 ( slightly high than the level
given ) asking treatment … I choose stop Ramipril

AfraTafreeh.com
Get more from: MplusX Qbank (https://member.mplusx.com/)
Facebook Page: (https://www.facebook.com/mplusxqbank/)

107. a patient presented with cough with greenish , blood streaked sputum for 2
days,… h/o night sweat, weightloss…. Lab given showing wbc – 35,000, blast cell
66%
Asking diagnosis..
a. acute leukaemia
b. CLL
c. Lung cancer
d. Metastasis
2. https://mrmjournal.biomedcentral.com/articles/10.1186/s4024
8-017-0107-2
3.
a.
108.35 years concerned about breast cancer … h/o breast cancer in maternal aunt
at 75 years … what will you do?
a. Advice about SBE
b. Usg
c. Mammogram
d. Tell to come when symptoms appear
109.a man come with pain and swelling of 1st MTP JOINT … what will confirm
diagnosis
a) MRI
b) XRAY
c) JOINT ASPIRATION

111. 9 YRS WITH HEAD INJURY WAS WITH HELMET …. Now GCS 15, playing
with ipad… FATHER wants to take her gaughter home .. what will you do
a) Sign DAMA and let her go
b) Tell father she must stay in hospital
112.lung cancer incidence increasing bur prevalence decreasing … what is the
reason
113.55 years old came for prostate cancer screening .. no symptoms of UTI..h/o
prostate cancer of father at 85 years … DRE normal… what will u do?
a) PSA

AfraTafreeh.com
Get more from: MplusX Qbank (https://member.mplusx.com/)
Facebook Page: (https://www.facebook.com/mplusxqbank/)

b) Trus
c) Tell him to come when symptoms appear
117. 75 yr old man living alone found to have increasing forgetfulness. Diagnosis?
A. cerebral infarct
B. cerebral tumor
C. cerebral abscess
D. cerebral hemorrhage
E. subdural haematoma

..
1. 9 months old child with URTI. He didn’t receive vaccines after 6 monthswhen developed thigh swelling after
receiving vaccine injections. Nowpresents with URTI and low grade fever. Advice about immunization schedule?
A. Re-commence primary immunization schedule
B. Give missing vaccines today
C. Give missing vaccines today and repeat after one month
D. Give missing vaccine today except pertussis and review in one month
E. After URTI resolves
2. Picture of a child with slapped cheek appearance.
Pregnant lady got exposed to this child. Best advice?
A. Assure the virus has no effect on fetus(i picked this, brain was not working)
B. Check maternal serum B19 antibodies
C. Review Hb after one month

AfraTafreeh.com
Get more from: MplusX Qbank (https://member.mplusx.com/)
Facebook Page: (https://www.facebook.com/mplusxqbank/)

D. Tell her to consult immediately if develops rash

3. Women 10 weeks pregnant her elder son 6 years old has chicken pox she did not have any chicken pox vaccination or immunoglobulins during
child hood.what 'll you do
a.check varicella serology b.give her immunoglobulin c. Vaccinate her.

4. pregnant pt at 20 weeks with hsv infection. What is treatment for this pt?
a) oral acyclovir
b)iv acyclovir
c)oral famciclovir
d)topical antiviral

5. 1 yr old boy crying and catching his left ear repeatedly. On PE both ear tympanic memb red , exudates present on tonsils. No ear discharge or
nasal discharge
a- Acute otitis media
b- Acute tonsillitis
c- Chronic otitis media

6. old man 60 years old who is a heavy smoker for 40 years comes to you with a swelling in the tonsillar region,O/E there is a mass
in the tonsillar pouch,what is your diagnosis ?

a. Nasopharyngeal cancer(not sure)


b. Lipoma
c. lymphoma
d. laryngeal cancer e. metastatic spread
7. old man complaining of confusion and drowsiness for 5 days. HTN on drugs. warfarin for AF. BP normal. CT given, Dx
asked?
a) subdural hematoma
b) intracerebral hge
c)meningioma

AfraTafreeh.com
Get more from: MplusX Qbank (https://member.mplusx.com/)
Facebook Page: (https://www.facebook.com/mplusxqbank/)

d) Middle cerebral artery infarct

8.CT scan of diverticulitis in elderly woman with scenario of fever, LIF pain, mild leukocytosis, history of constipation and peri colic abscess found. Most
appropriate next treatment?(CT given)
A. Broad spectrum antibiotics with gut rest
B. Prednisolone
C. CT guided drainage
D. Laparoscopic surgery

E. sigmoid resection
9.Patient with sudden abdominal pain. X-ray show dilated loops of small bowel obstruction and air in biliary tree.
Cholangitis
Cholelithiasis
Pancreatitis
Crohn’s ileitis

10.Woman has left groin mass. No trouble for her. On examination, discrete, tender, smooth surface 3cm mass is felt just
below and lateral to pubic tubercle. Which is appropriate?
A- Abdominal USG
B- CT abd and pelvis C- Surgical exploration D- Fine needle aspiration E- Core biopsy

11. Diabetic retinopathy sceanario and fundoscopy pic..asking diagnosis

AfraTafreeh.com
Get more from: MplusX Qbank (https://member.mplusx.com/)
Facebook Page: (https://www.facebook.com/mplusxqbank/)

121212 12.Hypertensive retinopathy scenario & funduscopy given. Asking cause of funduscopy picture

.
13.Molluscum contagious pic was given and asked for school exclusion:
a. when fever subside
b. when respiratory infection subside
c. 7 days after first rash
d. 4 days after antibiotics

e. when crusts dry


14.handbook pic of salivary stones-options are-
a.opg
b.usg
c.ct head

AfraTafreeh.com
Get more from: MplusX Qbank (https://member.mplusx.com/)
Facebook Page: (https://www.facebook.com/mplusxqbank/)

d.intraoral xray

16. A 30 years old woman with increasing dyspnea, dry cough over few months with painful lumpy skin lesions on legs and shins. On
examination, there are red, painful, lumpy lesions in leg and bilateral ankle swelling. ACE level increased. Which of the following is the
best next step to get diagnosis? (No given X-ray)
A. CT scan chest
B. Skin biopsy
C. Blood culture
D. Aspiration
E. ANA
17. 17 years old boy come with Gingival inflammation and swelling with submandibular lymph nodes enlargement,soft,tender.
next step?
A)CT head
B)OPG
C)gingival swab
D)FNAC E) blood culture (picked this one..dont know exact)

18. A man got back from a business trip to combodia,now presents with jaundice,fever,RUQ pain.he came back from trip 5days
ago,no rash.high billirubin , all the liver enzymes were so high.and low platelet.dx
1.hepatitis A
2.malaria
3.dengue
19. 20-year-old is involved in an RTA with multiple f/x and undergoes surgery. 12hrs post-surgery, patient is unresponsive but moans to the
pain. What do you do next?
a.6 L O2 by ventury mask

b. 2 L O2 via prongs

c.Send back to surgery

d.Reassure

e.Nasal cannula

20. Old px underwent surgery of femur fracture, 5 days after had dyspnea and confusion. On auscultation, there is crepitation in
right lower lobe.
A. Pulmonary Embolism(choose that.. read somewhere fat embolism occurs 24-74 hours)
B. fat embolism
C. pneumonia
D. aspiration syndrome

21. 35 yrs old male went to Pub, where a woman bite him, totally unknown and she went away. 2 cm lesion, bleeding
continues, teeth mark, he is fully tetanus immunized 3 yrs ago..what to do now
A.suture and review in 24 hrs

AfraTafreeh.com
Get more from: MplusX Qbank (https://member.mplusx.com/)
Facebook Page: (https://www.facebook.com/mplusxqbank/)

B.hep B vaccine and immunoglobulin


C.zidovudine
d. Flucloxacillin
E.azithromycin

23. 15 years old girl comes to u for OCP advice...few days later her mother also ur patient comes to u and ask about her
daughter's health issue.... what to do
1, make an appointment with daughter again
2, tell mother that u can’t tell anything
3, ask mother to discuss this with her daughter

24. Mother and daughter coming to your clinic for checkup and the daughter (17 years)asked privately to be tested for
chlamydia ..you took a swab and the result was negative
Second day mother called and asked about the results of her daughter
a)You tell the mother that test for chlamydia was negative
b) You tell the mother that you will discuss the results of her daughter privately with her
c) Ask the mother to bring the daughter and come for meeting
d) You tell that you can not disclose the information

25. A young man was admitted to ER for fracture. His sister accompanied him and she wanted you to write a letter to her
employer because she wanted to take a day off.
A. Write her the letter.
B. Refuse.
C. Write the letter stating that her brother was in an accident and she had to take care of him

d.write that she needs off because she is sick.

26. A party event was hosted where 20 people attended.


Food items Symptom Occurred Non-symptom No. of people
BBQ chicken 2 9 11
Cheese cake 10 2 12
Sandwich 10 8 18
Egg salad 7 10 17
Frozen berry smoothie 10 10 20
Which is the likely source of food poisoning?
A- BBQ chicken
B- Cheese cake
C- Sandwich
D- Egg salad
E- Frozen berry smoothie

27. 28-year-old female with history of multiple jail admissions and prostitution, and drug intakecomes to you for psychiatric evaluation from
the court. She tells you that about how she left school and home around 13 years oldto start her life. On examination, you see that she is
quiet but anxious and when you ask her to tell more about herself, she becomes chaotic and irritated. What is your diagnosis?
a) Antisocial
b) conduct disorder
c) borderline
d) drug abuse
28. 30 years old woman wants to conceive 12 months of
amenorrhea, bmi 30,previously irregular menstruation 5-
6 times a year. Lab values fsh 2(3.3-15), Lh 6(n),prolactin,
estrogen, tsh normal. What will help in Dx?
A.S.progesteron
B.S.testosterone
C.MRI pituitary
D.pelvic usg.
29. 18 y lady came with left lower abdominal pain.her menstrual cycle usually is for 4-6 weeks ,she has the same pain 4 weeks
before and she had her last menstruation 9 days ago.She usually has this pain 1 day before her menstruation starts and
continues for 3 days and pain increase with urination.What is the cause of her pain?

a. Ectopic pregnancy
b. Dysmenorrhea
c. complex corpus luteum cyst
d. torsion of ovary
e.ovulation
29. Patient BMI 32. Everything normal. Amenorrhoea for 12 month. Previously irregular (5-6 times yr). Now came for infertility
Fsh 2 (normal)
LH normal
Prolactin 450 ( upto 700) (was normal..can’t remember the unit,but it was upto 700 sure)
TSH Normal range
Oestradiol normal range
What inv to confirm

a.S.testosterone
b.usg

AfraTafreeh.com
Get more from: MplusX Qbank (https://member.mplusx.com/)
Facebook Page: (https://www.facebook.com/mplusxqbank/)

c.CT scan
d.Endometrial biopsy

30. 48 years old woman last menstruation was 8 month ago come with vaginal bleeding for last 3 days.. what is the most
common cause?
a)endometrial cancer
b)cervical cancer
c)vaginal atrophy
d)follicular activation
e)cervical glandular hyperplasia
31. 50yr lady came for screening, didn’t have any screening before.mother with cervical cancer 54,grandmother with breast
cancer at 60 or 70, what ll be best screening adv for her :
Colonoscopy
Colposcopy
Mamography
Cervical screening test(ans)

32. 47Patient on warfarin.. had PR bleeding. InR 9. Whats most appropiate next tx?
A whole blood tx
B packed cell tx
C ffp tx
D vit k
E cease warfarin
38. 3 years and 5 months old boy, BMI 18, growth chart given. Ask?
a. mildly overweight
b. he is on ideal weight
c. underweight

39. a 10 yr old child had seizure at home.after that he became unconscious and remain.he was brought to ed via
ambulance.blood glucose 2.wat will u give
a.im glucagon
b.iv dextrose
c.iv glucagon(

40.pt was on multiple drug, ace inhi, simvastin and so many. Asking cause. Choose simvastin..as it causes hyperkalemia(not sure)

41. Lady post operative.. Na120, other electrolytes normal ask test to diagnose?a. serum osmolality, b.urine osmolality, c.uribe
electrolytes

43. HT with CDM pt on ACEI, warfarin, metformin, statin, amiodaroen started 3 weeks ago. suddenly got swelling at Rt thigh for
2 days & become painful,tem 37.4. Rt thigh size is 4 cm larger than Lt. Dx ?
a.Rhabdomyolysis
b.Haematoma
c.DVT
d.Cellulitis

drug reaction was not in option

44. A scenario of a patient who underwent gastrectomy for carcinoma of stomach antrum developed pain and tenderness and
other serious symptoms (don’t remember). Values were given of which K+ was 6 and CK = 2500. Other values were normal.
Most likely diagnosis?
a. Rhabdomyolysis

45. old patient with decrease in vision from past few months .vision s 6/18 on right side and 6/12 on left and on correction with
pinhole is 6/9 on both sides .What is the cause fr this condition?
Retinal degeneration
Glaucoma
Cataract
refractive error
presbyopia

46. Patient on medications for HTN. she feels dizzy when get out of the bed.. happenining for a while.. she also had some fal ls.
BP 160/100
Has AF. Next Ix
A echo
B xray
C holter monitoring
D 24 hour BP monitoring
E check BP again in supine

AfraTafreeh.com
Get more from: MplusX Qbank (https://member.mplusx.com/)
Facebook Page: (https://www.facebook.com/mplusxqbank/)

48. 18 m Child crying and waking up at night everytime mother give fruit juice .Ur advice
A.add milk thickener to bottle
B.give cows milk
C.give water.
D.give bottle milk

49. Woman want to be pregnant on sodium valproate for epilepsy.wel controlled what to do?
A.dec the dose of Na valproate
B.chane to phenytoin
C.stop the med as she became good
D.leave her on the same dose

50.young female have amenorrhoea. All the hormone normal. Fsh ,LH,Prolaction ,even FSH:LH also normal..TSH 0.1(normal
level:0.4-0.6) given. Asked cause? I chose primary hyporthyroidism

51. 38 yrs old asking for COCP, she smokes 15 cigarettes, has family history of ovarian n breast cancer, she also has history of
premenstrual headaches what is most likely a contraindication/reason for not prescribing COCP to her
Her history of premenstrual headaches
family of ovarian cancer
Family ho breast cancer
Age
Smoking
52. A 54 year man presented with epigastric pain and vomiting and nausea for 12 hour duration.he has had this pain on and off for
last 3 month with loss of appetite and weight loss. Further more he is chronic alcoholic otherwise not significant history
onexamination his sclera is mild ecteric . ther is intrahepatic dilation only no intra hepaitic or extra hepatic bile duct obstr uction .lab
values given . whats most appropriate next?
Bilirubin 54
Alp 88
Ast
Ggt 155
Lipase 300
Normal values given can compare with given values
Ct abdomen (
Ercp
Mri abdomen
Endoscopic view

53. resident while doing laparoscopy causes injury to liver surgeon comes and control but it results in laparotomy what should be
done?
Tell the patient he made error by registrar
Tell the patient it was complementary complication by registrar
Don’t tell as patient is recovered
Surgeon have to tell complete history to patient(ans)
Surgeon have to tell to patient in the present of second person

54.Ultrasound confirms testicular mass. Alpha fetoprotein done.what else marker will do to confirm? Bhcg
55. abnormal behaviour old pt.. worsening alzeimer’s

56.old lady forgetfulness,foget all the kitchen staff, when ask become unfamiliar,did nt say anything, when going to do MMSE, didn’
t able to co operate. What else will lead u to diagnosis? a) anhedonia(i picked but may b hoarding is the ans) b)Hoarding
c)suspiciousness

57. fetal heart rate was 60 and after emergency LUCS,the heart rate remained 60 despite resuscitation.What maternal
knvestigation will help you with diagnosis.
Factor V Leiden
Lupus anticoagulant
Anti ro antibody
Protein C

58. 74 years old.pt with urgency,frequency and nocturia.on ex right lobe of prostate is enlarged and irregular. Psa done 20ng.
its repeated after two weeks. Value still same. What next should b done
a.turp
b. prostate biopsy
c.bone scane
d. pmsa pet

59.supraventricular tachycardia ecg. No symp.. only BP 90/60 mm (hg). What is the next most appropriate.? a) adenosine
bolus b)carotid sinus massage c)DC cardioversion d)verapamil.
60. Pharmaceutical company approaches u with results of comparison between 2 drugs for hyper cholesterolrmia, wat is d important
question u wil ask
A- wether same dose of dugs wer used
B- wether 2 grp s contained d same no of people
how did u equalise d dietary factors

AfraTafreeh.com
Get more from: MplusX Qbank (https://member.mplusx.com/)
Facebook Page: (https://www.facebook.com/mplusxqbank/)

61. There was a question about Forest Plot. Looks like this but the numbers at the base are 0.001, 0.01, 1, 10, 100. They
conducted a study about
The graph measures the presence and absence of pain. What does the plot mean?
a. DRUG X is more effective than Drug Y
b. DRUG Y is more effective than Drug X

63. a boy(6 years) keeps telling tummy ache when his father has to leave for job to stay overnight, sometimes doesnot want to get off
from the car when dropped school, but alright when he sit in the class.this becoming worse for last 2 weeks. What additional thing will
help diagnosis? a)bulling at school b) recent nocturnal enuresis chose this one as he is alright when he is at school.
64. Patient presents to you who has kept a knife for his protection. He submits the knife for security check but he is evasive
about keeping other weapons and refuses to allow you to talk to his psychiatrist. What will help u to find about collateral
history?
A. Ask his relative about his mental history
B. Ask to neighbor about his history
C. Ask police about the case
D. Override his refusal and talk to psychiatrist anyway
E. Wait until he can be persuaded to volunteer the information

68. Child after cold sores developed pin point lesions on trunk and limbs . lesions were unblanchable. Labs given. Clinical
examination were normal.
Hb- 86g/dl
WCC-normal
Platelets-35*10. Ask treatment. A.strick bed rest. B.prednosolone.
69. Young man, with hereditary spherocytosis (AD), asking how many of his siblings will be affected:
50% of all siblings
25% of all male as he already has
25% of all siblings
50% of all male

70. Similar left hand picture (distal amputed finger) cause?


a.Bronchiectasis
b.Pulmonary HTN
c. Emphysema
d.Pulmonary Infiltrate

AfraTafreeh.com
Get more from: MplusX Qbank (https://member.mplusx.com/)
Facebook Page: (https://www.facebook.com/mplusxqbank/)

e.Chronic asthma

71 71.wht symptom will u find most commonly in hereditary hypercholestrolemia?


a.Cornea arcus
b.Tendon xanthoma
c.orbital xanthoma
d.hepatomegaly

72. 7 years old boy with recurrent abcesses, each time positive staph aureus. On exam, pharyngitis n gingivitis.
What will u do to find the cause?
1. Neutrophil function
2. Lymphocyte function
3. Complement function

73. C

74. u are a resident, surgeon while doing laparoscopy causes injury to liver surgeon comes and control but it results in
laparotomy what should be done?
Tell the patient he made error by registrar
Tell the patient it was complementary complication by registrar Don’t tell as patient is recovered
Surgeon have to tell complete history to patient
Surgeon have to tell to patient in the present of second person
75. pt has to get morphine , but after 2 days still have severe pain.u checked and saw nurse gave only paracetamol. what will u
do next.? a. formal complaint to head nurse. B talk to nurse c. nurse supervisor d. Write down the incident

76. 60 yr lady with pain controlled on regular opioid analgesics come for a new prescription.2 weeks ago u gave her a
prescription of 2 month but she now comes and says the pharmacist made a mistake in dispensing the drugs.pharmacist
denied any mistake in dispensing.what will u do?
A.precribe her 2 weeks opioid
B.ask the pharmacist any record of dispensing
C.document the discrepancy between women and pharmacist saying
D.report abt women's telling lie

78. 45 years old man came to ask for the advice about the PSA screening in Prostate Malignancy as his grandfather diagnosed at
the age of 70 and his neighbor now diagnosed to have as he is now 65.What information will u give to him?
a. Screening is not indicated in this age ??
b. DRE is the good testing than PSA in this age
c. Can do now
d. It has high false positive rate
79. 37 wk pregnant lady with mild pre-eclampsia for induction of labour. Bishop
score 2.What will you do?
a.Elective LUCS
b.Induction with PGE 1 and ARM after 4 hours
c.Induction with PGE 2 and ARM after 12 hours.

AfraTafreeh.com
Get more from: MplusX Qbank (https://member.mplusx.com/)
Facebook Page: (https://www.facebook.com/mplusxqbank/)

d.Urgent LUCS
e.Syntocinon infusion

80.39 wk pregnant, CTG given normal, has no fetal movement for last 24 hours. Mx
a.Admission for urgent induction
b.Admission for observation
c.Reassure
d.Send home and check after 24 hours.
e.reassure and ask her to come in next visit

81. Pt posted for surgery on femur on multiple meds. Which should be stopped at least a week before
a. Ramipril
b. Metformin
c. Ibuprofen
d. Thiazides
e. Amitriptylin

82. Female patient whose husband died 2 weeks ago. she is having symptoms of says she looks her husband in
bed when she wakes up. during interview she cries and tearful what will tell her? A. Counsel about normal grief.b. tell
her brereavement/

83. Mother came with 17 years old son history of 6 years social withdrawal. He was excellent student
before.Doesn’t work, no hobby or social life,reluctant to talk .dx?
A.schizophrenia
B.depression
C.schizoaffective disorder
d.agarophobia

84. You are a doctor in a town, where 6 people out of 100 are non-smoker. What are the
chances of stroke. The chances of stroke in smoker is 50%more than non-smoker. Now
the pharmaceutical company is introducing a medicine which reduces the chances of
stroke by 1/3rd in smoker population. What is the percentage of the smoker population
will get stroke due to drug?
A. 4%
B. 6%
C. 9%
D. 12%
E. 20%

85. A 45-year-old male presented to the emergency department with left sided chest pain radiating to the left arm and
jaw. There is no ECG changes and troponin is negative 4 hours since the onset of chest pain. He has been on
sildenafil for erectile dysfunction, metformin for type 2 diabetes mellitus, and ramipril for hypertension. What would be
the most appropriate treatment?
a. Nitrates
b. Heparin
c. Morphin
d. Beta blocker
e. Paracetamol

86. Stress incontinence scenario ,,,,with cough or sneezing ....best next invest,
Ct
Urodynamic studies
Urine analysis
Cystoscopy

87. Old woman with urgency and nocturia for a long time. Now having incontinence for like a month. She couldn't
reach the toilet downstairs. She also has bilateral knee OA. What is the best management for her?
A. Fluid restriction at night
B. Install a camode in her bedroom.
C. Oxybutynin
D. Paracetamol

88. old man had Trans urethral prostatectomy, what will b the complication?
Retrograde ejaculation
Urine dribbling
Erectile dysfunction

AfraTafreeh.com
Get more from: MplusX Qbank (https://member.mplusx.com/)
Facebook Page: (https://www.facebook.com/mplusxqbank/)

89. kids Periorbital cellulitis,eyelid swelling,orbital redness ,fever,he can't move eyeball,there is marked tenderness
and erythema around the eye, most appropriate next investigation to diagnosis?
CT scan
Swab test
Blood culture

90.bacterial conjunctivitis scenario. Ask what tropical tx will give? A. Chloramphenicol.b.acetozolamide. c. Tamolol

91. screening : age 41.. only 1 relative have colon cancer at age 63. What to do? FOBT 2 yearly

(Can’t remember anymore.. please keep me in your prayers.. wish u all the best..)

1. CT abdomen of diverticulitis asking for dx and mx (aspiration/ broad spectrum


antibiotics and gut rest)
2. Child with meningitis, buldging anterior fontanelle, antibiotics given and culture taken,
what to give next? A. IV phenytoin B. IV Dexamethasone

3. Suspicious of nasopharyngeal carcinoma, what’s the best imaging? A. CT scan B. MRI C.


xray

4. Immigrant child (3yo) Mantoux test noted 12mm, whats the next mx? A. CXR B. start
isoniazid C. repeat Mantoux
5. Pt with submandibular swelling, afebrile, mobile mass, non-tender, what’s the best
imaging? A. sialogram B. xray C. CT scan
6. CTG of a 32 week pregnancy showing variable deceleration, immediate mx?
A. Observe B. repeat CTG in 24 hours C. C-sec D. advise hydration and repeat in 4
hours

7. Given low TSH, normal T4 value, a. subclinical hyperthyroidism b.


subclinical hyperthyroidism c. pituitary adenoma (can’t remember
other options)

AfraTafreeh.com

You might also like